You are on page 1of 250
»Complete and detailed explanation of the formulas and principles *Over S00 solved and supplementary problems Useful tables and conversion factor In the Appendix «With index for easy access to topics in the book *includes the recent board examination questions wy DIT GU LESANIA Uy Wl! DILLOOATLA www.gertcrev.com ut Mechanics & Hydraulics AbLtitis Ss DIEGO INOCENCIO T. GILLESANIA Civil Engineer BSCE, LIT — Magna Cum Laude 5th Place, PICE National Students’ Quiz, 1989 Awardee, Most Outstanding Student, 1989 3rd Place, CE Board November 1989 Review Director & Reviewer in all Subjects Gillesayia Engineering Review Center Reviewer in Mathematics and General Engineering Sciences MERIT Philippines Review, Manila Author of Various Engineering Books Fluid Mechanics and Hydraulics Revised Edition Copyright © 1997, 1999, 2003 by Diego Inocencio Tapang Gillesania All rights reserved. No part of this book may be reproduced, stored in a retrieval system, or transferred, in any form or by any means, without the prior permission of the author ISBN 971-8614-28-1 Printed by: GPP Gillesania Printing Press Ormoc City, Leyte Philippines Cover design by the author The cardinal objective of this book is to provide reference to Engineering students taking-up Fluid Mechanics and Hydraulics. This may also serve as a guide to engineering students who will be taking the licensure examination given by the PRC. The book has 9 chapters. Each chapter presents the principles and formulas involved, followed by solved problems and supplementary problems. Each step in the solution is carefully explained to ensure that it will be readily understood. Some problems are even solved in several methods to give the reader a choice on the type of solution he may adopt. To provide the reader easy access to the different topics, the book includes index. Most of the materials in this book have been used in my review classes, The choice of these materials was guided by their effectiveness as tested in my classes. I wish to thank all my friends and relatives who inspired me in writing my books and especially to my children and beloved wife Imelda who is very supportive to me. I will appreciate any errors pointed out and will welcome any suggestion for further improvement. i. DIEGO INOCENCIO T. GILLESANIA Cebu City, Philippines * To my mother Iluminada, my wife Imelda, and our Children Kim Deunice, Ken Dainiel, and Karla Denise TABLE OF CONTENTS ' Preface........ Dedication CHAPTER 1 Properties Of Fluid .......ccccceccee eset teeesessntesesseeeeeateeeenteneeneencnere if Types of Fluid... : Mass Density....... Specific Volume Unit Weight or Specific Weight «0... Specific Gravity ...:ccsceesecseeesesessesesrenseereeseens VISCOSIYiess. cs sbeletat- . Kinematic Viscosity .....c.ccccccscsssseseseseetsescesanesseserenensesasanereenens 5 Gurriace VOLSIOM:. cscecissdavsermes For an ideal gas, its density can be found from the specific gas constant and ideal gas law: Pp == Eq. 1-2 RT CHAPTER ONE 3 FLUID MECHANICS Properties of Fluids & HYDRAULICS where: p = absolute pressure of gas in Pa R= gas constant Joule / kg-°K For air: R = 287 J/kg-°K R=1,716 lb-ft/slug-"R T = absolute temperature in °Kelvin °K = °C + 273 °R = °F + 460 - Table 1 - 1: Approximate Room-Temperature Densities of Common Fluids Fluid pin kg/m? Air (STP) 1,29 Air (21°F, a 1tm) 1.20 Alcohol : 790 Ammonia 602 Gasoline 720 Glycerin 1,260 Mercury a2 13,600 Water 1,000 SPECIFIC VOLUME, V, : Specific volume, V., is the volume occupied by a unit mass of fluid. Vez Eq. 1-3 _ UNIT WEIGHT OR SPECIFI€ WEIGHT, y ae Specific weight or unit weight, 7, is the weight of a unit volume of a fluid. _ weight of fluid, WwW ie volume, V VSP S. FLUID MECHANICS & HYDRAULICS 4 CHAPTER ONE Properties of Fluids Units: English :. Ib/ft® Metric : dyne/cm? SI : N/m? or kN/m? SPECIFIC GRAVITY Specific gravity, s, is a dimensionless ratio of a fluid’s density to some standard reference density. For liquids and solids, the reference density is water at 4° C (39.2° F). Pliquid s= Hqul P water In gases, the standard reference to calculate the specific gravity is the density of air. g= ee Eq. 1-7 Pair 5 For water at 4°C: y = 62.4 lb/ft? = 9.81 kN/m3 p = 1.94 slugs/ft} = 1000 kg/m? s=1.0 VISCOSITY, 1 (MU) The property of a fluid which determines the amount of its resistance to shearing forces. A perfect fluid would have no viscosity. Consider two large, parallel plates at a small distance y apart, the space between them being filled with a fluid. Consider the upper plate to be subject to a force F so as to move with a constant velocity U. The fluid in contact with Lene fixed plate FLUID MECHANICS CHAPTER ONE 5 & HYDRAULICS Properties of Fluids the upper plate will adhere to it and will move with the same velocity U while the fluid in contact with the fixed plate will have a zero velocity. For small values of U and y, the velocity gradient can be assumed to be a straight line and F varies as A, U and y as: wh ACL E u Fe —— or —i«c — y An but Wiis av (from the figure) yy . = Shearing stress, t dV dV i & = OF t =k — dy dy where the constant of proportionality k is called the dynamic of absolute viscosity denoted as k. dV dy T= t: eee See Eq. 1-8 dV /dy a lu where: * t = shear stress in lb/ft? or Pa u = absolute viscosity in lb sec/ ft? (poises) or Pa-sec. y = distance between the plates in ft or m U = velocity in ft/s or m/s KINEMATIC VISCOSITY v (NU) Kinematic viscosity is the ratio of the dynamic viscosity of the fluid, 11, to its mass density, p. ¥ yet Eq. 1-9 where: = absolute viscosity in Pa-sec p = density in kg/m? CHAPTER ONE Properties of Fluids FLUID MECHANICS & HYDRAULICS 6 Table 1 - 2: Common Units of Viscosity System Absolute, p lb-sec/ ft? (slug/ft-sec) Kinematic, v English ft2/sec dyne-s/ cm? (poise) Pa-s (N-s/m?) cm2/s (stoke) m2/s Note: 1 poise = 1 dyne-s/cm? = 0.1 Pa-sée 1 stoke = 0.0001 m2/s % (1 dyne = 10° N) SURFACE TENSION o (SIGMA) The membrane of “skin” that seems to form on the free surface of a fluid is due to the intermolecular cohesive forces, and is known as surface tension. Surface tension is the reason that insects are able to sit on water and a needle is able to float on it. Surface tension also causes bubbles and droplets to take on a spherical shape, since any other shape would have more surface area per unit volume. Pressure inside a Droplet of Liquid: > Q Eq. 1-10 where: o = surface tension in N/m d = diameter of the droplet in m p = gage pressure in Pa FLUID MECHANICS CHAPTER ONE 7 & HYDRAULICS Properties of Fluids Capillarity (a) Adhesion: > Cohesion (b) Cohesion > adhesion Capillarity (Capillary action) is the name given to the behavior of the liquid in a thin-bore tube. The rise or fall or a fluid in a capillary tube is caused by surface tension ané depends on the relative magnitudes of the cohesion of the liquid and the adhesion of the liquid to the walls of the containing vessel. Liquids rise in tubes they wet (adhesion > cohesion) and fall in tubes they do not wet (cohesion > adhesion). Capillary is important when using tubes smaller than about 3/8 inch (9.5 mm) in diameter. _ 4ocos0 yd h For complete wetting, as with water on clean glass, the angle 0 is 0°. Hence the formula becomes n= 42 Eq. 1-12 ya where: l,= capillary rise or depression inm y = unit weight in N’/m? d = diameter of the tube inm o = surface tension iin Pa FLUID MECHANICS 3 CHAPTER ONE & HYDRAULICS Properties of Fluids Table 1 - 3: Contact Angles, 0 Materials Angle, 0 mercury-glass 140° water-paraffin -107° water-silver 90° kerosene-glass glycerin-glass COMPRESSIBILITY, B Compressibility (also known as the coefficient of compressibility) is the fractional change in the volume of a fluid per unit change in pressure in a constant- Temperature process, where; f AV = change in volume V = original volume Ap = change in pressure dV/V = change in volume (usually in percent) BULK MODULUS OF ELASTICITY, E, The bulk modulus of elasticity of the fluid expresses the compressibility of the fluid. It is the ratio of the change in unit pressure to the corresponding volume change per unit of volume. CHAPTER ONE 9 Properties of Fluids FLUID MECHANICS & HYDRAULICS stress Ap Fo.1-15 Pk q. strain = AV. 4 dp : Ep=- Eq. 1-16 CEE AY IY PRESSURE DISTURBANCES Pressure disturbances imposed on a fluid move in waves. The velocity or celerity of pressure wave (also known as acoustical or sonic velocity) is c= Ep le Eq. 1-17 p Bp PROPERTY CHANGES IN IDEAL GAS for any ideal gas experiencing any process, the equation of state is given by: expressed as: PiVi _ P2V2 Eq. 1-18 - es When temperature is held constant, Eq. 1 - 18 reduces to (Boyle’s Law) Eq.1-19 | When temperature is held constant (isothermal condition), Eq. 1 - 18 reduces to (Charle’s Law) * 2 Eq. 1- 20 eae | pi Vi =p V2 10 CHAPTER ONE FLUID MECHANICS Properties of Fluids 3 & HYDRAULICS For Adiabatic or Isentropic Conditions (no heat exchanged) pr Vik = pe Vat = Constant where: pi = initial absolute pressure of gas p2 = final absolute pressure of gas V, = initial volume of gas V2 = final volume of gas T) = initial absolute temperature of gas in °K (°K = °C + 273) T2 = final absolute temperature of gas in °K k = ratio of the specific heat at constant pressure to the specific heat at constant volume. Also known as adiabatic exponent. VAPOR PRESSURE Molecular activity in a liquid will allow some of the molecules to escape the liquid surface. Molecules of the vapor also condense back into the liquid. The vaporization and condensation at constant temperature are equilibrium | ' processes. The equilibrium pressure exerted by these free molecules is known as the vapor pressure or saturation pressure. Some liquids, such as propane, butane, ammonia, and Freon, have significant vapor pressure at normal temperatures. Liquids near their boiling point or that vaporizes easily are said to volatile liquids. Other liquids such as mercury, have insignificant vapor pressures at the same temperature. Liquids with low vapor pressure are used in accurate barometers. The tendency toward vaporization is dependent on the temperature of the liquid. Boiling occurs when the liquid temperature is increased to the point that the vapor pressure is equal to the local ambient (surrounding) pressure. Thus, a liquid’s boiling temperature depends on the local ambient pressure, as well as the liquid’s tendency to vaporize. PLUID MECHANICS CHAPTER ONE 11 & HYDRAULICS Properties of Fluids Table 1 - 4: Typical Vapor Pressures mercury [0.000173 turpentine 0.0534 water. ethyl alcohol ether butane Freon-12 propane ammonia Solved Problems Problem 1 - 1 A reservoir of glycerin has a mass of 1,200 kg and a volume of 0.952 cu. m find its (a) weight, W, (b) unit weight, y, (c) mass density, p, and (d) specific pravity (s). Solution « (a) Weight, V=Me = (1,200)(9.81) Weight, W = 11,772 N or 11.772 KN (b) Unit weight, y= < i 1772 0.952 Unit weight, y = 42.366 kN/mé M yi Density, = — (¢) ensity, p V 1200 0,952 Density, p = 1,260.5 kg/m' Density, p = 12 CHAPTER ONE Properties of Fluids FLUID MECHANICS & HYDRAULICS (d) Specific gravity, s= 8 P water 1,260.5 1,000 Specific gravity, s = 1.26 Specific gravity, s = Problem 1 - 2 The specific gravity of certain oil is 0.82. Calculate its (a) specific weight, in Ib/f8 and kN/m3, and (b) mass density in slugs/ft® and kg/m’, Solution (2) Specific weight, y = Ywater x $ Specific weight, y = 62.4 x 0.82 = 51.168 lb/ft? Specific weight, y = 9.81 x 0.82 = 8,044 kN/m3 (b) Density, p = pwater * $ Density, p = 1,94 x 0.82 = 1.59 slugs/ft3 Density, p = 1000 x 0.82 = 820 kg/m3 Problem 1 - 3 A liter of water weighs about 9.75 N. Compute its mass in kilograms. Solution Mass = me & Mass = a. 9.81 Mass = 0.994 kg Problem 1 - 4 If an object has a mass of 22 kg at sea level, (a) what will be its weight at a point where the acceleration due to gravity g = 9.75 m/s?? (b) What will be its mass at that point? c CHAPTER ONE 1D MECHANICS : Mi rsitauiics Properties of Fluids 1 3 Solution (a) W=mg = 22(9.75) W=214.5N (b) Since the mass of an object is absolute, its mass will still be 22 kg Problem 1-5 What is the weight of a 45-kg boulder if it is brought to a place where the acceleration due to gravity is 395 m/s per minute? Solution W= Me Imin = 305 m/s s mir min 60sec g = 6.583 m/s? W = 45(6.583) W = 296.25 N Problem 1 - 6 lf the specific volume of a certain gas is 0.7848 m?/kg, what is its specific Weight? Solution p = 1.2742 kg/m? ‘ Specific weight, y= p xg = 1.2742 x 9.81 Specific weight, y =12.5 N/m? FLUID MECHANICS & HYDRAULICS 14 CHAPTER ONE Properties of Fluids Problem 1-7 What is the specific weight of air at 480 kPa absolute and 21°C? Solution = where R = 287 J/kg-°K _ _ 480x10° 287(21 + 273) p = 5.689 kg y= 5.689 x 9.81 y= 55.81 N/m? Problem 1 - 8 Find the mass density of helium at a temperature of 4 °C and a pressure of 184 kPa gage, if atmospheric pressure is 101.92 kPa. (R = 2079 J/kg * °K) Solution ; Pp D p= eee ensity pP P = Peage + Patm = 184 + 101,92 p = 285.92 kPa P44 273= 277°R 285.92. 10° 2,079(277) Density, p = 0. 4965 kg/m$ Density, p = Problem 1-9 \t 32°C and 205 kPa gage, the specific weight of a certain gas was 13.7 N/m’. | etermine the gas constant of this gas. FLUID MECHANICS & HYDRAULICS CHAPTER ONE 15 Properties of Fluids Solution Density, p = ae § _ 137 9.81 = 1.397 kg/m? nn Density, p = —— y,P RT > 3 jengy os cates WE laae) 10" Note: pam = 101.325 kPa R(32 + 273) Gas constant, R = 718.87 J/kg - °K Problem 1 - 10 Air is kept at a pressure of 200 kPa absolute and a temperature of 30°C in a 500-liter container. What is the mass of air? Solution at RT 200 x 10° p=23 kg/m? p= Mass =p x V = 500. 2.3% ao0 Mass = 1.15 kg Problem 1- 11 A cylindrical tank 80 cm in diameter and 90 cm high is filled with a liquid. The tank and the liquid weighed 420 kg. The weight of the empty tank is 40 kg. What is the unit weight of the liquid in kN/m’. 16 CHAPTER ONE Properties of Fluids FLUID MECHANICS & HYDRAULICS Solution -M Re = ahs = 840 kg/m? 4 i - VPS = 840(9.81) = 8240.4 N/m} y = 8.24 kN/m3 Problem 1 - 12 A lead cube has a total mass of 80 kg. What is the length of its side? Sp. gr. of lead = 11.3. Solution Let L be the length of side of the cube: M=pV 80 = (1000 x 11.3) L3 L=0.192 m=19.2 cm Problem 1 - 13 A liquid compressed in a container has a volume of 1 liter at a pressure of 1 MPa and a volume of 0.995 liter at a pressure of 2 MPa. The bulk modulus of elasticity (Eg) of the liquid is: Solution < rdPe sae oat ~dV/V— (0.995=1)/1 Eg = 200 MPa Problem 1 - 14 What pressure is required to reduce the volume of water by 0.6 percent? Bulk modulus of elasticity of water, Es = 2.2 GPa. _ FLUID MECHANICS CHAPTER ONE 17 & HYDRAULICS Properties of Fluids Solution i dP dV/V dp = pr- pi pi =0 dp = pa dV= V2 3 Vy dV = -0.6% V = -0.006V Es = E,= dt Ned PD ae 5 0.006V /V p2 = 0.0132 GPa p2 = 13.2 MPa x ¢ Problem 1-15 Water in a hydraulic press, initially at 137 kPa absolute, is subjected to a pressure of 116,280 kPa absolute. Using Ez = 2.5 GPa, determine the percentage decrease in the volume of water. Solution sade dV/V i} ee 3 15x10 =- (116,280 — 137) x 10 dV/V aes -0.0465 V dV — = 4.65'% decrease V Problem 1 - 16 . If 9 m3 of an ideal gas at 24 °C and 150 kPaa is compressed to 2 m%, (a) what is the resulting pressure assuming isothermal conditions. (b) What would have been the pressure and temperature if the process is isentropic. Use k = 1.3 CHAPTER ONE 19 UID MECHANICS Properties of Fluids # HYDRAULICS CHAPTI 1 8 Prone er| bles Ber eHanicg & HYDRAULICS Solution (a) For isothermal condition: Pi V= P2 V2 150(9) = pa (2) p2 = 675 kPa abs Two large plane surfaces are 25 mm apart and the space between them is filled with a liquid of viscosity 1 = 0.958 Pa-s. Assuming the velocity gradient to be # straight line, what force is required to pull a very thin plate of 0.37 m? area at ‘constant speed of 0.3 m/s if the plate is 8.4 mm from one of the surfaces? (b) For isentropic process: i 7 Vik pee Solution 150(9)!3 = po (2)13 P= F, + Fe po = 1,060 kPa abs ent /y (kK-1)/k t | Po = FLA e Ps U/y me «pLTA T, Cee - — ——_— = aces’ y 24 : ae 150 0.958(0,3)(0.37) Tz = 466.4°K or 193.4°C FAi=——go166 = o4N ‘ re 0.958(0.3) (0.37) 5 enc Ea oe Pe POT ae en OG OROD BANS AGA a EM ee OE = =12. Problem 1 - 17 = 0.0084 &6.N If the viscosity of water at 70 °C is 0.00402 poise and its specific gravity is 0.978 BiG Ae Oe 5 F=19.06 N determine its absolute viscosity in Pa - s and its kin tic € ty m/s : ema viscosity in m2? and in stokes. ; Problem 1 - 19 A cylinder of 125 mm radius rotates concentrically inside a fixed cylinder of 130 mm radius. Both cylinders are 300 mm long. Determine the viscosity of Solution Absolute viscosity: = 0.00402 poise x 21 Fa—s lpoise the liquid which fills the space between the cylinders if a torque of 0.88 N-m is Lt = 0.000402 Pa - s fequired to maintain an angular velocity of 21 radians/sec Assume the velocity gradient to be a straight line Kinematic viscosity: y= He 0.000402 _ p (1000x 0.978) v= 411x107 m/s + lstoke v= 4.11 x 107 m2/s« —=— 0.0001 m?/s v = 4,11 x 10% stoke FLUID MECHANICS 2 0 CHAPTER ONE & HYDRAULICS Properties of Fluids Solution fixed £ aN -- y= 0.005m u/y U=ro U = 0.125(2z) rotating U = 0.785 m/s oe y= 0.005 m Torque = F(0.125) Torque = tA (0.125) 0.88 = t [27(0.125)(0.3)] (0.125) Tt = 29.88 Pa L=0.3m (ie fixed cylinder liquid 0.005 a nt 0,125 0.13 m ___ 29,88 4 0.785 /0.005 = 0.19 Pa-s Problem 1 - 20 An 18-kg slab slides down a 15° inclined plane on a 3-mm-thick film of oil with viscosity 1. = 0.0814 Pa-sec. If the contact area is 0.3 m?, find the terminal velocity of the slab. Neglect air resistance. Solution W = 18(9.81) = 176.58 N slab is Y Terminal velocity is attained when the sum of all forces in the direction of motion is zero. ; FLUID MECHANICS CHAPTER ONE 2 1 & HYDRAULICS Properties of Fluids [ZE, = 6] Wsin@- F,=0 F,=W sin 0 F, = 176.58 sin 15° (R=tAS goat y 176.58 sin 15° = 0.0814 4 (0.3) 0.003 Ul =5.614 m/s 0, = 5.614 m/s Problem 1 - 21 Estimate the height to which water will rise in a capillary tube of diameter 3 mm, Use o = 0.0728 N/m and y = 9810 N/ m+? for water. Solution Note: 0 = 90° for water in clean tube Capillary rise, h = a yd 4(0.0728) 9810(0.003) Capillary rise, = 0.0099 m = 9.9 mm Capillary rise, h = Problem 1 - 22 Estimate the capillary depression for mercury ina glass capillary tube 2 mm in diameter. Use o = 0.514 N/m and 0 = 140°. Solution 4docos@ _ 4(0.514)(cos 140°) yd (9810 x 13.6)(0.002) Capillary rise, h = -0.0059 m (the negative sign indicates capillary depression) Capillary rise, h = Capillary depression, li = 5.9 mm E CHAPTER ON 2 3 CHAPTER ONE FLUID MECHANICS Properties uf Fluids 22 UID MECHANICS Properties of Fluids & HYDRAULICS Problem 1 - 23 What is the value of the surface tension of a small drop of water 0.3 mm in diameter which is in contact with air if the pressure within the droplet is 561 Pa? “sonar transmitter operates at 2 impulses per second. If the device 1s held to © surface of fresh water (Ex = 2.04 x 10° Pa) and the echo is received midway {ween impulses, how deep is the water? Solution lution = 40 \ velocity of the pressure wave (sound wave) is s d . |Es 561 = —22 ee 0.0003 pope 5 = 0.042 N/m Z 9 transmitter Sound wave be oe = 1,428 m/s bie sei Wit ih ace % ; ) e vice the echo is received § Problem 1 - 24 / h i é e i lses, then An atomizer forms water droplets 45 4m in diameter. Determine the excess iidlway pees aor d 2 ressure within these droplets using o = 0.0712 N/m © total time of travel of sound, 8 ; » 14(0.5) = % sec and the total Solution distance covered is 2/1, then; 0 | thect d 2h = 1,428(4) = 40.0712) _ 6 359 Pa = 178.5 m 45107 Problem 1 - 25 Distilled water stands in a glass tube of 9 mm diameter at a height of 24 mm. What is the true static height? Use o = 0.0742 N/m. At what pressure will 80 °C water boil? {Vapor pressure of water at 80°C = 47 4 kPa) Solution Solution Water will boil if the atmospheric pressure equals the vapor piessure _ 40cosé yd where @ = 0° for water in glass tube Therefore water at 80 °C will boil at 47.4 kPa 4(0.0742) h= —\ 1 = 0.00336 m= 3.36 mm 9810(0.009) » True static height = 24 - 3.36 True static height = 20.64 mm 24 CHAPTER ONE © Properties of Fluids FLUID MECHANICS & HYDRAULICS [Supplementary Problems Problem 1 - 28 What would be the weight of 1 3- mass or re! ere cceleratior b , g) 3-kg a plar wh he accel 0. Ans: 30 N Problem 1 - 29 ae ae tank with a diameter of 12 m and a depth of 4 m is filled er to the top with water at 20°C. [f the w i E : ter is heated to 50°C, } much water will spill over? Uni i : 5979 ? t weight of A FC i kN/m3 and 9.69 kN/m3, respectively. SA cae e Ans: 4.7 m3 Problem 1 - 30 A ee ae container is partially filled with a liquid at 15 atm. The volume of ee i a ee L. At a of 30 atm, the volume of the liquid ‘is : average bulk modulus of elasticity of the liqui 8iven range of pressure if the tem: ae ge of perature after compression j return to its initial value. Whatis the coefficient of empresa oe Ans: Eg = 1.872 GPa; B = 0.534 GPa"! Problem 1 - 31 Calculate the density of water : ne Bik Pou/iene vapor at 350 kPa abs and 20°C if its gas constant Ans: 2.59 kg/m Problem 1 - 32 Air is kept at a pressure of 200 kPa and a t Pot container. What is the mass of the air? Pee fee oe Ans: 1.15 kg CHAPTER ONE Properties of Fluids 2 5 ‘FLUID MECHANICS & HYDRAULICS Problem 1 - 33 (a) If 12 m3 of nitrogen at 30°C and 125 kPa abs is permitted to expand jsothermally to 30 m3, what is the resulting pressure? (b) What would the pressure and temperature have been if the process had been isentropic? Ans: (a) 50 kPa abs (b) 34.7 kPa abs; -63°C Problem 1 - 34 A square block weighing 1.1 kN and 250 mm on an edge slides down an incline on a film of oil 6.0 um thick, Assuming a linear velocity profile in the vil and neglecting air resistance, what is the terminal velocity of the block? The viscosity of oil is 7 mPa-s. Angle of inclination is 20°. Ans: 5.16 m/s Problem 1 - 35 Hlenzene at 20°C has a viscosity of 0.000651 Pa-s. What shear stress is required to deform this fluid at a strain rate of 4900 s?? Ans: t= 3.19 Pa Problem 1 - 36 A shaft 70 mm in diameter is being pushed at a speed of 400 mm/s through a bearing sleeve 70.2 mm in diameter and 250 mm long. The clearance, assumed uniform, is filled with oil at 20°C with v = 0.005 m2/s and sp. gr. = 0.9. Find the force exerted by the oil in the shaft. Ans: 987 N Problem 1 - 37 Two clean parallel glass plates, separated by a distance d = 1.5 mm, are dipped ina bath of water. How far does the water rise due to capillary action, if o = 0,0730 N/m? ‘ Ans: 9.94 mm FLUID MECHANICS & HYDRAULICS 26 CHAPTER ONE Properties of Fluids Problem 1 - 38 Find the angle the surface tension film leaves the glass for a vertical tube immersed in-water if the diameter is 0.25 inch and the capillary rise is 0.08 inch. Use o = 0.005 lb/ft. i a Ans: 64,3° Problem 1 - 39 ba force is required to lift a thin wire ring 6 cm in diameter from a water surface at 20°C? (o of wat PC = i ta ( water at 20°C = 0.0728 N/m). Neglect the weight of the Ans: 0.0274 N | & HYDRAULICS FLUID MECHANICS CHAPTER TWO 2 7 Principles of Hydrostatics Chapter 2 Principles of Hydrostatics UNIT PRESSURE OR PRESSURE, p Pressure is the force per unit area exerted by a liquid or gas on a body or surface, with the force. acting at right angles to the surface uniformly in all directions. In the English system, pressure is usually measured in pounds per square inch (psi); in international usage, in kilograms per square centimeters (kg/cm?), or in atmospheres; and in the international metric system (SI), in Newtons per square meter (Pascal). The unit atmosphere (atm) is defined as a pressure of 1.03323 kg/cm? (14.696 Ib/in?), which, in terms of the conventional mercury barometer, corresponds to 760 mm (29.921 in) of mercury. The unit kilopascal (kPa) is defined as a pressure of 0.0102 kg/emé (0.145 Ib/sq in). PASCAL'S LAW Puscal’s law, developed by French mathematician Blaise Pascal, states that the pressure on a fluid is equal in all directions and in all parts of the container. In Figure 2 - 1, as liquid flows into the large container at the bottom, pressure pushes the liquid equally up into the tubes above the container. The liquid rises to the same level in all of the tubes, regardless of the shape or angle of the tube. 28 CHAPTER TWO FLUID MECHANICS . Principles of Hydrostatics & HYDRAULICS -PLUID MECHANICS CHAPTER TW/O 29 & HYDRAULICS Principles of Hydrostatics ABSOLUTE AND GAGE PRESSURES Gage Pressure (Relative Pressure) Gage pressures are pressures above or below the atmosphere and can be Measured by pressure gauges or manometers., For small pressure differences, a U- fube manometer is used. It consists of a U-shaped tube with one end connected to _ the container and the other open to the atmosphere. Filled with a liquid, such as Water, oil, or mercury, the difference in the liquid surface levels in the two Manometer legs indicates the pressure difference from local atmospheric fonditions. For higher pressure differences, a Bourdon gauge, named after the Trench inventor Eugéne Bourdon, is used. This consists of a hollow metal tube with an oval cross section, bent in the shape of a hook. One end of the tube is tlosed, the other open and connected to the measurement region. Figure 2 ~ 1: Illustration of Pascal’s Law The laws of fluid mechanics are observable in many everyday situations. For : example, the pressure exerted by water at the bottom of a pond will be the same as the pressure exerted by water at the bottom of a much narrower pipe, provided depth remains constant. If a longer pipe filled with water is tilted so that it reaches a maximum height of 15 m, its water will exert the same pressure as the other examples (left of Figure 2 - 2). Fluids can flow up as well as down in devices such as siphons (right of Figure 2 - 2), Hydrostatic force causes water in the siphon to flow up and over the edge until the bucket is empty or the suction is broken. A siphon is particularly useful for emptying containers that should not be tipped. Atmospheric Pressure & Vacuum _ Atmospheric Pressure is the pressure at any one point on the earth's surface from the Weight of the air above it. A vactium is a space that has all matter removed from it. It is impossible to create a perfect vacuum in the laboratory; no matter how advanced a vacuum system is, some molecules are always present in the vacuum area, Even remote regions of outer space have a small amount of gas. A vacuum ran also be described as a region of space where the pressure is less than the Hormal atmospheric pressure of 760 mm (29.9 in) of mercury. Under Normal conditions at sea level: Patm = 2166 Ib/ ft? = 14,7 psi = 29.9 inches of mercury (hg) = 760 mm Hg = 101.325 kPa Absolute Pressure Absolute pressure is the pressure above absolute zero (vacuum) Pym Pam Py Figure 2 - 2: Illustration of Pascal’s Law Pabs = Pgage + Patm Eq. 2-2 Note: * Absolute zero is attained if all air is removed. It is the lowest possible pressure attainable. * Absolute pressure can never be negative. * The smallest gage pressure is equal to the negative of the ambient atmospheric pressure, 3 0 CHAPTER TWO FLUID MECHANICS Principles of Hydrostatics & HYDRAULICS bie Standard 60 gage 58.675 gage atmosphere-= 101.325 abs == 8 eee ~41.325 gage -40 gage 60 abs or -100 gage All pressure units in kPa Figure 2 - 3: Relationship between absolute and gage pressures Note: Unless otherwise specified in this book, the term pressure signifies gage pressure. MERCURY BAROMETER A mercury barometer is an accurate and relatively simple way to measure changes in atmospheric pressure. At sea level, the weight of the atmosphere forces mercury 760 mm (29.9 in) up a calibrated glass tube. Higher elevations yield lower readings because the atmosphere is less dense there, and the thinner air exerts less pressure on the mercury. vacuum 760 mm i Atmospheric pressure ANEROID BAROMETER In an anercid barometer, a partially evacuated metal drum expands or contracts in response to changes in air pressure. A series of levers and springs translates the up and down movement of the drum top into the circular motion of the pointers along the aneroid barometer's face. metal drum (partial vacuum) Hairspritig Current atmosphere = 100 abs Absolute zero = -101.325 gage CHAPTER TW/O Principles of Hydrostatics 31 “FLUID MECHANICS #& HYDRAULICS IARIATIONS IN PRESSURE se Consider any two points (1 & 2), whose difference in elevation i h, e = bay Bends i i -sectional area a and a leng) "ends of an elementary prism having across-sec! ae Bince this prism is at rest, all forces acting upon it must be in equilibrium ; Free liquid surface Dy & pz are gage pressures 7 Figure 2 - 4: Forces acting on elementary prism : Note: Free Liqufd Surface refers to liquid surface subject to zero gage r atmospheric pressure only. pressure or with With reference to Figure 2 - 4: W=yV W-=y (aL) (EE, = 0] F,- F;=Wsin® a-pia=y (aL) sin® a entane but Lsin® =h po- p= yh Epa: ints i 2 uid Therefore; the difference in pressure between any tivo ponte ina oe na at fest is equal to the product of the unit weight of the fluid (y) to the vertica (h) between the points. FLUID MECHANICS 3 2 CHAPTER TWO & HYDRAULICS Principles of Hydrostatics Also: po =pitwh Eq. 2-4 This means that any change in pressure at point 1 would cause an equal change at point 2. Therefore; a pressure applied at any point in a liquid at rest is transmitted equally and undiminished to every other point in the liquid. Let us assume that point ® in Figure 2 - 4 lie on the free liquid surface, then the gage pressure py is zero and Eq. 2 - 4 becomes: This'means that the pressure at any point “h” below a free liquid surface is equal to the product of the unit weight of the fluid (y) and h. Consider that points ® and @ in Figure 2 - 4 lie on the same elevation, such that ht = 0; then Eq. 2 - 4 becomes: Pi= pr Eq. 2-6 This means that the pressure along the same horizontal plane in a homogeneous fluid at rest are equal. Pressure below Layers of Different Liquids Air, pressure = py hi Liquid 1 he Liquid 2 hg FLUID MECHANICS & HYDRAULICS CHAPTER TWO 33 Principles of Hydrostatics Consider the tank shown to be filled with liquids of different densities a with air at the top under a gage pressure of pa, the pressure at the bottom o the tank is: Pootiom = YN + p= yl Ya he + ys his + pa Eq.2-7 PRESSURE HEAD Pais Pressure head is the height “h” of a column of homogeneous liquid of unt weight y that will produce an intensity of pressure p. To Convert Pressure head (height) of liquid A to liquid B YA Itg = ha SA or hp = in 2A or hg = ha —— 5B PB YB To convert pressure head (height) of any liquid to water, just multiply its height by its specific gravity ; Itwater = Miquid * Stiquid Eq: 2-10 CHAPTER TWO 34 FLUID MECHANICS Principles of Hydrostatics MANOMETER A manometer is a tube, usually bent in a form of a U, containing a liquid of known specific gravity, the surface of which moves proportionally to changes of pressure. [t is used to measure pressure Types of Manometer Open Type - has an atmospheric surface in one leg and is capable of measuring gage pressures Differential Type - without an atmospheric surface and capable of measuring only differences of pressure. Piezometer - The simplest form of open manometer. It is a tube tapped into a_ wall of a container or conduit for the purpose of measuring pressure. The - fluid in the container or conduit rises in this tube to form a free surface Limitations of Piezometer: ° Large pressures in the lighter liquids require long tubes Gas pressures can not be measured because gas can not form a free surface (b) Differential manometer (a) Open manometer (c) Piezometer & HYDRAULICS © D MECHANICS CHAPTER TWO 35 HYDRAULICS Principles of Hydrostatics ps in Solving Manometer Problems: 1. Decide on the fluid in feet or meter, of which the heads are to be expressed, (water is most advisable). 2 Starting from an end point, number in order, the interface of different fluids. 4, Identify points of equal pressure (taking into account that for a homogeneous fluid at rest, the pressure along the same horizontal plane are equal). Label these points with the same number. A 4, Proceed from level to level, adding (if going down) or subtracting (if going up) pressure heads as the elevation decreases or increases, — respectively with due regard for the specific gravity of the fluids. olved Problems “Problem 2 - 1 Ta depth of liquid of 1 m causes a pressure of 7 kPa, what is the specific “vity of the liquid? lution . Pressure, p= yh 7 = (9,81 x s) (1) s=0.714 > Specific Gravity H iblem 2 - 2 Whatis the pressure 12.5 m below the ocean? Use sp. gr. = 1.03 for salt water. slution p=yh p = (9.81 x1.03)(12.5) p= 126.3 kPa 36 CHAPTER TWO FLUID MECHANICS FLUID MECHANICS CHAPTER TW/O 37 Principles of Hydrostatics & HYDRAULICS & HYDRAULICS Principles of Hydrostatics Problem 2 - 3 lf the pressure 23 meter below a liquid is 338.445 kPa, dsenninetG uni weight y, mass density p , and specific gravity s. Problem 2-5 If the pressure in the air space above an oil (s = 0.75) surface in a closed tank is 115 kPa absolute, what is the gage pressure 2 m below the surface? Solution Solution P = Psurface # ¥ h Psurface = 115 - 101.325 Note: Patm = 101.325 kPa Psurface = 13.675 kPa gage p= 13.675 + (9.81x0.75)(2) (a) Unit weight, y p=yh 338.445 = y (23) y = 14.715 kN/m3 = 28, «°) Mass density, p p = 28.39 kPa hiesh : Problem 2 - 6 = 14.715 «10° Find the absolute pressure in kPa at a depth of 10 m below the free surface of 9.81 p = 1,500 kg/m! oil of sp. gr. 0.75 if the barometric reading is 752 mmHg. Solution (c) Specific gravity, s Pats = Patw * Pg its = Pat. + Page — Prluid Patm = Yn Hm Pwate = (9.81 x 13.6)(0.752) <= 2e00 Pam = 100.329 kPa ’ 1,000 Pabs = 100.329 + (9.81 x 0.75)(10) s=15 Pats = 173.9 kPa F : Problem 2 - 4 Problem 2 - 7 If the pressure at a point in the ocean is 60 kPa, what is the pressure 27 meters Belge th oy A pressure gage 6 m above the bottom of the tank containing a liquid reads 90 elow this poin kPa. Another gage height 4 m reads 103 kPa. Determine the specific weight of the liquid : Solution The difference in pressure between any two points ina Solution liquid is p2- py, =yh p2= pit yh p-pi=yh = 60 + (9.81x1.03)(27) p2 = 332.82 kPa 103 - 90 = y(2) * y = 6.5 kN/m3 38 CHAPTER TWO . FLUID MECHANICS Principles of Hydrostatics FLUID MECHANICS & HYDRAULICS CHAPTER TWO & HYDRAULICS Principles of Hydrostatics 39 Problem 2 - 8 Solution An open tank contains 5.8 m of water covered with 3.2 m of kerosene (y = 8 Since the density of the mud varies with depth, the pressure kN/m3). Find the pressure at the interface and at the bottom of the tank. should be solved by integration Solution dp=ydh (a) Pressure at the interface dp = (10 + 0.5 h)dh pa = Yk hu E ; : = (8)(3.2) [x = fo +0.5h)dh pa = 25.6 kPa Kerosene 5 . 5 (b) Pressure Te bottom re aNn p= 10h+ 0.25h2 =5 a a + yh Wate go es i . 5816 Bee a 1 =.9.81 kim? [eee : = [10(8) + 0.25(5)2] - 0 =9, ; 3 : = 56.25 kPa pa = 82.498 kPa r Problem 2-11 Problem 2-9 If atmospheric pressure is 95.7 kPa and the gage attached to the tank reads 188 {i the figure shown, if the atmospheric mmHg vacuum, find the absolute pressure within the tank. pressure is 101.03 kPa and the absolute essure at the bottom of the tank is “111.3 kPa, what is the specific gravity Solution , of olive oil? Povs = Pui * Pgage : ' Pgage = Ymercury mercury , = (9.81 x 13.6)(0.188) = 25.08 kPa vacuum Pgage = -25.08 kPa Pubs = 95.7 + (-25.08) Pavs = 70.62 kPa abs Problem 2 - 10 Solution The weight density of a mud is given by y = 10 + 0.5h, where y is in kKN/m? and Gage pressure at the bottom of the tank, p = 231.3 - 101.03 his in meters. Determine the pressure, in kPa, at a depth of 5m Gage pressure at the bottom of the tank, p = 130.27 kPa [p= Zyh] ] P= Yu lint Yo Ho + Yar Hw + Yoit Hout 130.27 = (9.81 x 18.6)(0.4) + (9.81 ~ s)(2.9) + 9.81(2.5) + (9.81 x 0.89)(1.5) 5 = 1.38 40 CHAPTER TWO FLUID MECHANICS Principles of Hydrostatics & HYDRAULICS Problem 2 - 12 If air had a constant specific weight of 12.2 N/m? and were incompressible, what would be the height of the atmosphere if the atmospheric pressure (sea level) is 102 kPa? ’ Solution Height of atmosphere, h = @ ¥ =, 102 10° 12.2 Height of atmosphere, hi = = 8,360.66 m Problem 2 - 13 (CE Board May 1994) Assuming specific weight of air to be constant at 12 N/m3, what is the approximate height of Mount Banahaw if a mercury barometer at the base of the mountain reads 654 mm and at the same instant, another barometer at the ‘top of the mountain reads 480 mm. Solution Poot - Prop = yh (Ym Hm)bottom — (fm Nm) top = (y hair (9,810 x 13.6)(0.654) - (9,810 x 13.6)(0.48) = 12h h = 1,934.53 m CHAPTER TW/O 4 1 FLUID MECHANICS Principles of Hydrostatics & HYDRAULICS Problem 2 - 14 Compute the barometri pressure at sea level is 1 R = 287 Joule /kg-°K. c pressure in kPa at an altitude of 1,200 mY if the 01.3 kPa, Assume isothermal conditions a 21 ease Solution For gases: dp = -pg dh ayaeie . PRT ? | ~ 987(21 + 273) p = 0,00001185 p dp = -(0.00001185 p)(9.81) dh 4P _ 90001163 dh | ‘ P 1200 ; a = -0,0001163 a 101.3x10" 0 | p 1200 | = —0,0001163h | | wy | 101.3x10° 0 | In p - In (101.3 x 10°) = - 0.0001163(1200 - 0) In p = 11.386 pe e386 p = 88,080 Pa CHAPTER TWO Principles of Hydrostatics FLUID MECHANICS & HYDRAULICS 42 Problem 2 - 15 Convert 760 mm of mercury to (a) oil of sp. gr. 0.82 and (b) water. Solution z S a: mercury (a) Noi Nercury ee Soil hig = 12.605 m of oil (6) Thwater = Fmmercury Smercury = 0.76(13.6) Hwater = 10.34 m of water Problem 2-16 (CE Board May 1994) A barometer reads 760 mmHg and a pressure gage attached to a tank reads 850 cm of oil (sp. gr. 0.80). What is the absolute pressure in the tank in kPa? Solution ; Pavs = Pam + Pyage = (9.81 x 13.6)(0.76) + (9.81 x 0.8)(8.5) Pavs = 168.1 kPa abs Problem 2 - 17 A hydraulic press is used to raise an 80-kN cargo truck. If oil of sp. gr. 0.82 acts on the piston under a pressure of 10 MPa, what diameter of piston is required? Solution Since the pressure under the piston is uniform: Force = pressure x Area 80,000 = (10 x 10%) 4 D? D=0.1 m=100mm CHAPTER TWO MECHANICS. : Fr Principles of Hydrostatics 4 HYDRAULICS 43 oblem 2 - 18 (CE November 1998) ton A has a cross-section of 1,200 sq. cm while that of piston B is 950 sq. cm. jth the latter higher than piston A by 1.75 m. If the intervening passages are lled with oil whose specific gravity is 0.8, what is the difference in pressure een A and B. H ‘Solution P PA~ Pp = Yo Ito = (9,810 x 0.8)(1.75) | Pa- ps = 13,734 Pa 1200 cm? 950 cm? Wrobiem 2-19 the figure shown, Welermine the weight W that can be carried by the , kN force acting on the Piston. 300 mm @ Solution Since points 1 and 2 lie on the same elevation, pi = p2 fon: £(0.03)* -#(0.3)° W=150 kN FLUID MECHANIC CHAPTER TWO 44 & HYDRAULI Principles of Hydrostatics Problem 2 - 20 A drum 700 mm in diameter and filled with water has a vertical pipe, 20 mn in diameter, attached to the top. How many Newtons of water must b poured into the pipe to exert a force of 6500 N on the top of the drum? Solution Force on the top: F=p-x Area 6500 = p x 4 (700? - 20?) p = 0.016904 MPa p = 16,904 Pa [p=yh] 16,904 = 98101 h=1,723m Area Weight = y x Volume = 9810 x 4 (0.02)2(1.723) Weight = 5.31 N Area on top 700 mm @ Problem 2 - 21 The figure shown shows a setup with a vessel containing a plunger and a cylinder. What force F is required to balance the weight of the cylinder if the weight of the plunger is negligible? Plunger, a = 0,00323 m? Oil, s = 0,78 Oil, s = 0.78 oO CHAPTER TW 45 ee Principles of Hydrostatics & HYDRAULICS Plunger, a = 0.00323 m Oil, s = 0.78 Oil, s = 0.78 [po - pr = Yo Ht) E F A 000ss3 pr = 309.6 F (kPa) Ww 44 A ~ 0.323 pn = 136.22 kPa 1 = 3S 136.22 - 309.6 F = (9.81 x 0.78)(4.6) F = 0.326 kN = 326 N Problem 2 - 22 aa The hydraulic press shown is filled with oil with sp. gr. 0.82. Neglecting ¥ weight of the two pistons, what force F on the handle is required to suppor the 10 kN weight? 400 mm CHAPTER TWO 46 Principles of Hydrostatics Solution Since points 1 and 2 lie on the same elevation, then; pi= pa fi oes Aye da BLO Ei PERE as x r 2 (0.075)> = (0.025) Fy =141KN [= Mo = 0] F(0.425) = F,(0.025) F(0.425) = 1.11(0.025) F = 0.0654 kN F=65.4N CHAPTER TWO Principles of Hydrostatics 47 FLUID MECHANICS & HYDRAULICS FLUID MECHANICS & HYDRAULICS Since the gage reads “FULL” then the reading is equivalent to 30 cm of gasoline Reading (pressure head) when the tank contain water = (y + 274g) cm of gasoline Yy 2s 30 y = 27.06 cm Then; Problem 2 - 24 (CE Board November 2000) flor the tank shown in the Figure, hy = 3m and hh, = 4m of In Determine the value 25mm 400 mm Fa FBD of the lever arm Problem 2 - 23 The fuel gage for a gasoline (sp. gr. = 0.68) tank in a car reads proportional to its bottom gage. If the tank is 30 cm deep an accidentally contaminated with 2 cm of water, how many centimeters of gasoline does the tank actually contain when the gage erroneously reads “FULL”? Solution Solution Summing-up pressure head from 1 to3 in meters of water Pa 4 nyo.g4) -x= F% ye Y 0 + 0.84 In - (4-3) = 0 hyp = 119m “Full” CHAPTERTWO — Gun 4g CHAPTER Two FLUID MECHANICS 9) yup NHECHAINICS incipteb of Fiydréstaties Principles of Hydrostatics & HYDRAULICS SHYDRAULICS ein Problem 2 - 25 (CE Board May 1992) In the figure shown, what is the static pressure in kPa in the air chamber? ; ‘ VW besod 33) Problem 2 - 26 (£605 isdirisvo 6 For the manometer Shown)! ayworde tolomonsrr on) fry determine the pressure at the center of the pipe. Mercury, s = 13.55 Oil, s = 0.80 noite Solution The pressure in the air space equals the pressure on the surface of oil, Ps Solution : 42 Sum-up pressure: heat Honk lto3in ipeters re water | Pisa. = Yu Nu =9.81(2)_ pr = 19.62 kPa Po - Pa = Yo Ne 19.62 - p3 = (9.81 x 0.80)(4) pr = -11.77 kPa p= = 144.7 kPa Another solution. Sum-up pressure head from 1 to 3 Pu ¥ in meters of water +2-4(0.80) = 23 yi 0+ 28392 22 9.81 ps = -11.77 kPa CH 5 0 IAPTER TXW/O Pidmeiples ofHydiastatics FLUID MECHANICS & HYDRAULICS Problem 2 - 27 (CE Board November 2001) Determine the value of y in the manometer shown in the Figure ao Air, 5 KPa w B} 3 Ir Solution 7 Summing-up pressure head from Air, 5 KPa A to B in meters of water: uw 3 A = + 3(0.8) +15 - (13.6) = 28 Y wW Ss 5 —— +3.9- BRE: 981 ! 13.6y ; = fs where pg = 0 y = 0.324 m FLUID MECHANICS & HYDRAULICS Problem 2 - 28 (CE May 1993) In the figure shown, when the funnel is empty the water surface is at point A and the mercury of sp. gt. 13.55 shows a deflection of 15 ent Determine the new deflection of mercury when the funnel is filled with water to B. Solution Figure (a): Level at A Solve for y in Figure (a): CHAPTER TWO 5 1 Principles of Hydrostatics 80 cm Figure (b): Level at B Sum-up pressure head from A to 2in meters of water: PA 4 y-0.15(13.55) = ry, Y 0+y-203=0 y = 2.03 m CHAPTER TWO 53 UID MECHANICS | Principles of Hydrostatics 52 CHAPTER TW/O FLUID MECHANICS DRAULICS Principles of Hydrostatics & HYDRAULIC Sum-up pressure head from 2 to m in meters of water: In Figure (8): When the funnel is filled with water to B, point 1 will move down to 1 ee = Pm ; t : : =| +7(13.6)-24= — with the same value as point 2 moving up to 2 Y Y 13.6y-x= Eq. (1) Sum-up pressure head from B to 2’: In Figure (b): ‘ Sum-up pressure head from 2’ to m’ in meters of water: Pat e +08 +ytx- (x +015 +2)(13.55) = 22 i Po’ + 0.2sin0 + y + 0.2)(13.6) - (x + 0.2) = 2 Y 1 0 + 0.80 + 2.03 + x - 27.1x - 2.03 =0 26.1 x = 0,80 Spot Spur wee 0+2.72sin 0 +13.6y + 2.72- x-0.2= 28 New reading, R = 15 + 2x = 15 + 2(3.1) 13.6y - x = 8,183 - 2.72 sin 0 Eq. (2) New reading, R = 21.2 cm bse hoe 3.6y - x = 13.6y - x 8.183 - 2.72 sind = 2 Problem 2 - 29) ieee pe eee The pressure at point m in the figure oo shown was increased from 70 kPa to 105 kPa. This causes the top level of Woblem 2 - 30 mercury to move 20 mm in the sloping tube, What is the inclination, 0? tlosed cylindrical tank contains 2 m of water, 3 m of oil (s = 0.82) and the air hove oil has a pressure of 30 kPa. If an open mercury manometer at the 8 Mercury om of the tank has 1 m of water, determine the deflection of mercury. ©) J Air, 30 kPa tion Sum-up pressure head from 45 1 to 4 in meters of water: 3m 2m 1m Solution Pair 4. 300,82) +2+1-y(13.6) = 4 if te ot, +2.46+3-13.6y=0 y = 0.626 m joa 0. 8 Figure (a) Figure (b) In Figure (a): CHAPTER TWO Principles of Hydrostatics FLUID MECHANICS. & HYDRAULICS 54 Problem 2 - 31 The U-tube shown is 10 mm in diameter and contains mercury. If 12 ml of water is poured into the right-hand leg, what are the ultimate heights in the two legs? ee 120 mm os Solution Solving for h, (see figure b): 2 Volume of water = 4 (22) h = 12cm h = 15.28 em = 152.8 mm Note: 1 ml = 1 cm? Since the quantity of mercury before and after water is poured remain the same, then; i 120(3)=R+x+120+. R+2x=240 > Eq. (1) Figure (a) Figure (b) for gage reading of -17.1 kPa, Plezometer columns E, F, and CHAPTER TW/O 1D MECHANICS | Principles of Hydrostatics DRAULICS In Figure (6): Summing-up pressure head from 1 to 3 in mm of water: Fi 4 152.8 - (13.6) = -2 Ne yi R=11.24 mm In Eq. (2): 11.24 + 2x = 240 x= 114.38 mm Ultimate heights in each leg: Right-hand leg, hr =h +x = 152.8 + 114.38 Right-hand leg, hg = 267.18 mm Left-hand leg, hp = R + x = 11.24 + 114.38 Left-hand leg, hy, = 125.62 mm ~ oblem 2 - 32 termine the (#) elevations of «© liquids in the open and (b) the deflection of the Mercury in the U-tube Manometer neglecting the Weight of air. CHAPTER TWO 56 Principles of Hydrostatics Ee DRA Solution Gage + GL 15m _ 3m E El. 12m | | h 4 El l2m 2 4m oom ‘ 4m | | Pi = Pa =-17.1 kPa 4 Elam he J ‘Mercury, § = 13.6 Column E Sum-up pressure head from 1 toe in metes of water. 1a ae es Y Y Bet + (0.7) = 0 hy =2.5m Surface elevation = 15 - hy Surface elevation = 15-2.5=12.5m Column F Sum-up pressure head from 1 to fin meters of water; Fi 3107) joey = © ¥ ¥ ae T2119 =0 hy = 0.357 m Surface elevation = 12 + In Surface elevation = 12 + 0.357 = 12.357 m FLUID MECHANICS & HYDRAULICS Solution CHAPTER TWO 57 Principles of Hydrostatics Column G Sum-up pressure head from 1 to g in meters of water; PL 4300.7) +40) -In(1.6) = Y ¥ ay, = AZL +214+4-1.6h3=0 hy =2.72m Surface elevation = 8 + ls Surface elevation = 8 + 2.72 = 10.72 m Deflection of mercury Sum-up pressure head from 1 to 5 in meters of water; Pr 49¢07) +44 4-m(13.6) = PE Y Y =17: 4a + 10.1 - 13.6l4 ‘Iu = 0.614 m : Problem 2 - 33 An open manometer attached to a pipe shows a deflection of 150 mmHg with the lower level of mercury 450 mm below the centerline of the pipe carrying water. Calculate the pressure at the centerline of the pipe. Sum-up pressure head from 1 to Sin meters of water; PL 40.45 -0.15(13.6) = ¥ Y Pi_ 49.45-2.04=0 981 p= 15.6 kPa TW HAPTER TWO 58 SHAPTERTWO FLUID MECHANICS: FLUID MECHANICS Raine oar yarostenres 59 Principles of Hydrostatics & HYDRAULICS & HYDRAULICS ~ rincip! Problem 2 - 34 For the configuration shown, calculate the (a) Gage liquid = mercury, h=01m weight of the piston if the pressure gage - head from reading is 70 kPa. Sum-up pressure 1 to 4 in meters of water; +x +h-h(13.6)-x-15= a =1,5-0.1 + 0.1(13.6) 2.76 m of water == |= = (Ss =|> 1 ee ae Solution Sum-up pressure head from A to B in meters of water; Gage liquid = carbon tetrachloride wer reading, h =? oe (0.86) = ug Sum-up pressure head from 1 to 4in meters of water; : Br gy +h-h(159)-x-15= PAL 096= 2208 y 9.81 9.81 Ps pa = 78.44 kPa re reek 154 0.59h Weight = Fa where es _ P4 2276m > from (a) = pa x Area ry: A = 78.44 x = (1)? oe + 0.591 = 2,136 m Weight = 61.61 kN i roblem 2 - 36 Problem 2 - 35 : In the figure iawn, determine Air, p = 175 kPa abs Two vessels are connected to a differential manometer using mercury, the the height 1: of water and the connecting tubing being filled with water. The higher pressure vessel is 1.5 ‘ hage reading at A when the lower in elevation than the other. (a) If the mercury reading is 100 mm, what “absolute pressure at B is 290 is the pressure head difference in meters of water? (b) If carbon tetrachloride (s = 1.59) were used instead of mercury, what would be the manometer reading for the same pressure difference? CHAPTER TW/O Principles of Hydrostatics 60 Solution Sum-up absolute pressure head : Air, p = 175 kPa abs from B to 2 in meters of water; Fe .0.7(13.6)-h= 22 . 20 .952-n= 1B h= 2.203 m Sum-up absolute pressure head from B to A in meters of water; 2b) -o7aae)yeo7 = 24. Y y 290 i GPA oat - 9.52 + OF S81 pa = 203.5 kPa abs Problem 2 - 37 In the figure shown, the atmospheric pressure is 101 kPa, the gage reading at A is 40 kPa, and the vapor pressure of alcohol is 12 kPa absolute. Compute x + y. Solution Sum-up absolute pressure head from 1 to 2 in meters of water; - - (0.9) = s 40+101 _ ae ey [= 9.81 9.81 y=1461m FLUID MECHANIC & HYDRAULICS CHAPTER TW/O FLUID MECHANICS Principles of Hydrostatics & HYDRAULICS Sum-up pressure (gage) head from 1 to 4 in meters of water; Pa 4 x(0,9) +1.3(0.9)-1.3(13.6) = 2 : — +0.9x- 16.51 =0 9.81 x=13.81m Then, x + y = 28.42 m Problem 2 - 38 For the manometer setup shown, determine the difference in pressure between A and B. Solution x+0.68=yt17 x-y=1,02m * > Eq. (1) Sum-up pressure head from A toB in meters of water; PA _~.0.68(0.85) + y= P i Pa _ PB -y_y+0578 > Eq. (2) Y ai Substitute x - y= 1.02 in Eq. (1) to Eq. (2): Pa _ PB —102+0.578 ee. PA>PB ~4508 9.81 pa-ps = 15.68 kPa CHAPTER TW/O 6 3 Principles of Hydrostatics 62 CHAPTER TWO ‘FLUID MECHANICS Principles of Hydrostatics & HYDRAULIC! 4 HYDRAULICS Problem 2 - 39 A differential manometer is attached to a pipe as shown. Calculate the — pressure difference between points A and B. the figure shown, the leflection of mercury is initially 750 mm. If the pressure at A is Wiereased by 40 kPa, while Maintaining the pressure at B ‘yonstant, what will be the new ‘Wereury deflection? Mercury Solution - Solution Sum-up pressure head from A to B in meters of water; PA é —' Bp — - y(0.9y - 0.1(13.6) + 0.1(0.9) + (0.9) = —— PAbook ¥ ¥ = 0.1(13.6) - 0.1(0.9) 7A — PB PA EBs Bp ey 981 pa - pe= 1246 kPa Figure (a) Figure (b) In Figure a, sum-up pressure head from A to B in meters of water; PB PA _ 0.6 -0,25(13.6) + 0.25 + 21= : Pa _.P® = 4.65 m of water ip ¥ 6 4 CHAPTER TWO Principles of Hydrostatics In Figure b, pa’ = pat 40 Sum-up pressure head from A ‘to B in meters of water; (0.6- x) - (0.25 + 2x)13.6 + (2.35 + x) = PA ¥. pn, +40 " Pa Peer a 34297 5r to abs em PB: PA OU es a5 opie AB y 981 . ? i \ > EA ED e082 2.458 But PA . EB 7 65 i! i ¥ ¥ 1.65 = 25.2 x - 2.423 x = 0.162 m = 162 mm New mercury deflection = 250 + 2x = 250 + 2(162) New mercury deflection = 574 mm Problem 2 - 41 In the figure shown, determine the difference in pressure between points A and B. Kerosene, s = 0.82 FLUID MECHANIC. & HYDRAULIC ‘HYDRAULICS PLUID MECHANICS CHAPTER TW/O 65 Principles of Hydrostatics plution ne Kerosene, s = 0.82 s = 0.0012 Sum-up pressure head from A to B in meters of water; PA + 0,2(0.88) - 0.09(13.6) - 0.31(0.82) + 0,25 - 0.1(0.0012) = 78 Y Y PA _ PB _ 1.0593 m of water Y if pa - Po = 9.81(1.0523) = 10.32 kPa Problem 2 - 42 (CE Board) Assuming normal barometric pressure, how deep in the ocean is the point where an air bubble, upon reaching the surface, has six times its volume than ‘it had at the bottom? Solution eeaproreeeere Applying Boyle’s Law (assuming isothermal condition) ° ° Fa Vi= p2 V2] h pi = 101.3 + 9.81(1.03)h : ° pi = 101.3 + 10.104 h Wea V 2 p2=101.3+0=1013 le V2 =6V 7 ° (101.3 + 10.104) V = 101.3 (6 V) 4 10.104 h = 101.3(6) - 101.3 h= 50.13 m 66 SHAPTERTWo FLUID MECHANIC ‘MECHANICS CHAPTER TWO 67 Principles of Hydrostatics & HYDRAULIC VYORAULICS Principles of Hydrostatics Problem 2 - 43 f ince the pressure in air inside the tube is uniform. “then pr = po = 20.0124 kPa Pe = Yw ht 20.0124 = 9.81h; h=2,.04m A vertical tube, 3 m long, with one end closed is inserted vertically, with 4 open end down, into a tank of water to such a depth that an open manomet connected to the upper end of the tube reads 150 mm of mercury. Neglectir vapor pressure and assuming normal conditions, how far is the lower end § the tube below the water surface in the tank? Then, x=h+y=2.04+ 0.495 x= 2.535 m Soluti Hen Area=A blam 2 - 44 bottle consisting of a cylinder 15 cm in diameter and 25 cm high, has a neck is 5 cm diameter and 25 cm long. The bottle is inserted vertically in , with the open end down, such that the neck is completely filled with Find the depth to which the open end is submerged. Assume normal mmetric pressure and neglect vapor pressure. on plying Boyle’s Law Applying Boyle’s Law: ae pi Vir pe Ve Jore the bottle was inserted Volume of air: Vy = © (159? (25) + £ (5)? (25) V, = 4,908.74 cm* Before the tube was inserted; Absolute pressure of air inside, pi = 101.3 Volume of air inside, V; =3A Absolute pressure tn air: When the tube was inserted; pr = 101.325 Absolute pressure of air inside, po = 101.3 + 9.81(13.6)(0,15) Absolute pressure of air inside, p) = 121.31 kPa Volume of air inside the tube, V2 = (8 - y)A 1) the bottle is inserted: Volume of air: Va= 4 (15)? (25) ya ye | V2 = 4,417.9 cm? em Cae ee sure in Bi 3-y=2. f : y = 0.495 m p2 = 101.325 + 9.81 h [pi Vi = p2 Va] 101.325(4,908.74) = (101.325 + 9.81 h)(4.417 9) 101.325 + 9.81 lr = 112.58 h=1.15 cm r x=h+ 25= 26.15 cm From the manometer shown; Po = Yn Mn = (9.81 x 13.6)(0.15) pr = 20.0124 kPa. : 68 CHAPTER TWO Principles of Hydrostatics Problem 2 - 45 A bicycle tire is inflated at sea level, where the atmospheric pressure is 10] kPaa and the temperature is 21 °C, to 445 kPa. Assuming the tire does n expand, what is the gage pressure within the tire on the top of a mounta where the altitude is 6,000 m, atmospheric pressure is 47.22 kPaa, and tl temperature is 5 °C; Solution PiYi = PV Te ie At sea level: Absolute pressure of air, p: = 101.3 + 445 Absolute pressure, p, = = 546.3 kPaa Volume of air, V; = V Absolute temperature of air. T; = 21 + 273 = 294 °K On the top of the mountain: Absolute pressure of air, p) = 47,22 + p Since the tire did not expand, volume of air, V, = V Absolute temperature of air, T2=5 + 273 = 278 °K i T, 546.3(V) _: (47.22 + p\V GOAn| hore 47.22 + p = 516.57 p = 469.35 kPa FLUID MECHANIC & HYDRAULIC CHAPTER TW/O ‘MECHANICS i v Betics Principles of Hydrostatics 69 pplementary Problems lem 2- 46 Weather report indicates the barometric pressure is 28.54 inches of mercury. i rei r re inch? s the atmospheric pressure in pounds per square inc ; - ‘ : Ans: 14.02 psi iblem 2-47 lube shown is filled with oil. Determine the pressure heads at B and Cin ders of water. Ans: Ee = -2.38 m Af Pe = 951m Y Oil, s = 0.85 Oil, s = 0.85 lem 2 - 48 the tank shown in the figure, compute the pressure at points B, C, D, and E Ps. Neglect the unit weight of air. Ans: pr = 4.9; pc = po = 4.9; pe = 21.64 JO HAPTER Two FLUID MECHANICS 1 CHAPTER TWO 69 Principles of Hydrostatics & HYDRAULIC! Principles of Hydrostatics Problem 2 - 49 A glass U-tube open to the atmosphere at both ends is shown. If the U-tub contains oil and water, determine the specific gravity of the oil Ans: 0.8 ather report indicates the barometric pressure is 28.54 inches of mercury. iit is the atmospheric pressure in pounds per square inch? Ans: 14.02 psi lem 2 - 47 » (ibe shown is filled with oil. Determine the pressure heads at B and Cin ers of water. Ans: Pe =-2.38 m mM PC 2.051m Problem 2 - 50 i 2 ie A glass 12 cm tall filled with water is inverted. The bottom is open. What i r the pressure at the closed end? Barometric pressure is 101.325 kPa. Ans: 100.15 kPag - | Oil, s = 0.85 Problem 2 - 51 In Figure 13, in which fluid will a pressure of 700 kPa first be achieved? Oil, s = 0.85 Ans: glycerit Po = 90 kPa ) the tank shown in the figure, compute the pressure at points B, C, D, and E 1 Ma. Neglect the unit weight of air. ethyl alcohol 60m Ans: pe = 4.9; pe = po = 4.9; pe = 21.64 » = 773.3 ka/m? oil p = 899.6 kg/m? uo water p = 979 kg/m? glycerin p = 1236 kg/m? 70 pt Two } FLUID MECHA\ 1D MECHANICS CHAPTER TWO 71 rinciples of Hydrostatics & HYDRAULIC HYDRAULICS t Principles of Hydrostatics Problem 2 - 49 iblem 2 -~ 52 lindrical tank contains water at a height of 55 mm, as shown. Inside is a all open cylindrical tank containing cleaning fluid (s.g. = 0.8) at a height /r. i) pressure ps = 13.4 kPa gage and pc = 13.42 kPa gage. Assume the cleaning iil is prevented from moving to the top of the tank. Use unit weight of lor = 9.79 kKN/nx’. (a) Determine the pressure pa in kPa, (b) the value of Ji in m, and (c) the value of y in millimeters. A glass U-tube open to the atmosphere at both ends is shown. If the U-tul contains oil and water, determine the specific gravity of the oil Ans: 0: Ans: (a) 12.88; (b) 10.2; (c) 101 55mm Problem 2 - 50 A glass 12 cm tall filled with water is inverted. The bottom is open. What the pressure at the closed end? Barometric pressure is 101.325 kPa. Ans: 100.15 kPa Mercury (s.g. =:13.6) Problem 2 - 51 In Figure 13, in which fluid will a pressure of 700 kPa first be achieved? Ans: i ns: ely roblem 2 - 53 differential manometer shown is measuring the difference in pressure two Water pipes. The indicating liquid is mercury (specific gravity = 13.6), In is 675 m, Jini is 225 mm, and Jt. is 300 mm. What is the pressure differential ‘tween the two pipes. Po = 90 kPa ethyl alcohol 60m » = 773.3 kg/m? Ans: 89.32 kPa ol » = 899.6 kg/m? 10am water p = 979 kg/m? hy glycerin 5 p = 1236 kg/m? Pe | | 72 CHAPTER TWO FLUID MECHANI MECHANICS : CHAPTER nie 73 Principles of Hydrostatics & HYDRAULI RAULICS Total Hydrostatic Force on Surfaces piston which fits into a cylinder having a diameter of 60 mm. What fore acts on the larger piston, if the volume between C and D is filled with wate Ans: 15.83] al Hydrostatic Force Surfaces Problem 2 - 54 A force of 460 N is exerted on lever AB as shown. The end B is connected HYDROSTATIC FORCE ON PLANE SURFACES pressure over a plane area is uniform, as in the case of a horizontal submerged in a liquid or a plane surface inside a gas chamber, the }iydrostatic force (or total pressure) is given by: Problem 2-55 An open tube open tube is attached to a tank as shown. If water rises to height of 800 mm in the tube, what are the pressures p, and pp of the air abot water? Neglect capillary effects in the tube, pis the uniform pressure and A is the area. © ease of an inclined or vertical plane submerged in a liquid, the total ye can be found by the following formula: Ans: pa = 3.92 kPa; pr = 4.90 kl ’ Liquid surface Ss Center of gravity, cg Center of pressure, cp % ~ 1: Forces on an inclined plane CHAPTER THREE 74 FLUID MECHANIC Total Hydrostatic Force on Surfaces IECHANICS CHAPTER THREE 75 & HYDRAULIC y ULICS Total Hydrostatic Force on Surfaces a Consider the plane surface shown inclined at an angle @ with the horizonti To get the total force F, consider a differential element of area dA. Since th element is horizontal the pressure is uniform over this area, then; ON OF F (yp): e 3 - 1, taking moment of force about S, (the intersection of the ation of the plane area and the liquid surface), Py = [ree where dF=yysin@dA F=ysin@ Ay dF = pda where p= yh p=yysinGg dF =yysin@ dA [ir =ysin0 [i dA From calculus, [rw = AG) © yin 8 AG yp = [vtysinoaa) yain0 Ay yp =y.sin 0 fv dA F=ysin@ Ay From calculus, |r dA =Is (moment of inertia about S) F=y(¥sin0) A ; : AY Yp = 1s 4 From the figure, 7 sin@ = ji Then, Is eel ae Eq. 3-4 Yp AY ; y transfer forrhula of moment of inertia: Since yi is the unit pressure at the centroid of the plane area, Peg the formu Is=1,+ AY? may also be expressed as: [ ee , ys -—=— F=pyA Eq. 3-3 AY Gna 7 y= Y + — Eqg.3-5 Eq. 3 - 2 is convenient to use if the plane is submerged in a single liquid an AY without gage pressure at the surface of the liquid. However, if the plane submerged under layers of different liquids or if the gage pressure at thi so : = + e 3-1, then liquid surface is not zero, Eq. 3 - 3 is easier to apply. See Problem 3 - 15 ees 9 Ory Tigue ‘ et : Eq. 3-6 Eccentricity, e= —— q.o- cel ty, AY 76 CHAPTER THREE FLUID MECHAN| I) MECHANICS CHAPTER THREE 77 Total Hydrostatic Force on Surfaces & HYDRAUL VWORAULICS Total Hydrostatic Force on Surfaces Half ellipse Quarter elli ipse TABLE 3 - 1: Properties of Common plane sections a a a et a v1 i ke b/2 fe b/2 —| anneal Ix = 0.11 ab? = arb : Area = bd ns ade bd? db? aba? : = ae = — = core Sab) = % Area = 2bh Sa y= 5 ley Tpe = 0.055203 Ip, = 0,055ba ‘bad 3 ee ive 8 | Secwrofacice—d Parabolic segment Area = & bh 3 i eh Hey 4r ReneS Ye 3 Area =%4 7r2; x.= y= — a 3x Area = mr = 4 7D? vr iD! 4 64 ar? 16 | lex = Igy = 0.05574 L=h= Spandrel Segment of arc : Area = ab Length of are = r(20) = 2r0 4r Area = art, y= — 3 rsin@ en I nab Area = L bh ae n+1 a 5] a+. When 6 = 90° (semicircle) x= Ete ht 4n+2 ine 2r nr 78 CHAPTER THREE : FLUID MECHANIC MECHANICS ; : CHAPTER THREE 79 Total Hydrostatic Force on Surfaces & HYDRAULI RAULICS Total Hydrostatic Force on Surfaces TOTAL HYDROSTATIC FORCE ON CURVED SURFACES CASEI: FLUID IS ABOVE THE CURVED SURFACE. Fu = peg A or Fu=yhA Fy=yV tan 0 = Fy/Fy re; A © vertical projection of submerged curve (plane area) pry = pressure at the centroid of A Curved surface ile The procedure used in solving Fy is the same are that presented in Page 73. Vertical projection of the curved surfac WII; FLUID BELOW AND ABOVE THE CURVED SURFACE CASE II: FLUID IS BELOW THE CURVED SURFACE Volume = V cg of volume Wate projection’ of the curved surfa¢ Curved surface Net verteal Net vertical force te | projection of area 8 O CHAPTER THREE FLUID MECHANI : JID MECHANICS CHAPTER THREE 81 Total Hydrostatic Force on Surfaces & HYDRAULIC DRAULICS Total Hydrostatic Force on Surfaces DAMS Dams are structures that block the flow of a river, stream, or other waterway) Upstream Some dams divert the flow of river water into a pipeline, canal, or channe ee Others raise the level of inland waterways to make them navigable by shi Pownstream and barges. Many dams harness the energy of falling water to generate electri power. Dams also hold water for drinking and crop irrigation, and provid flood control. PURPOSE OF A DAM Dams are built for the following purposes: 1. Irrigation and drinking water 2. Power supply (hydroelectric) 3. Navigation ea 4, Flood control 5 Multi purposes Vigure 3 - 3: Boat Passing through Canal Lock. Canal locks are a series of gates designed fo allow a boat or ship to pass from one level of water to another. Here, after a boat has ‘entered the lock and all gates are secured, the downstream sluices open and water flows through them. When the water level is equal on either side of the downstream gate, water stops flowing through the sluices; the downstream gate opens, and the boat continues on at the new water level. Reservoir Powerhouse » TYPES OF DAMS 1. Gravity dams use only the force of gravity to resist water pressure — that is, they hold back the water by the sheer force of their weight pushing downward. To do this, gravity dams must consist of a mass | : nuh so heavy that the water in a reservoir cannot push the dam a me Ne / downstream or tip it over. They are much thicker at the base than the ; top—a shape that reflects the distribution of the forces of the water against the dam. As water becomes deeper, it exerts more horizontal pressure on the dam. Gravity dams are relatively thin near the surface Draft tube Penstock Bedrock of the reservoir, where the water pressure is light. A thick base enables the dam to 44withstand the more. intense water pressure at the bottom of the reservoir. Figure 3 - 2: Section of a dam used for hydroelectric 82 ua: CHAPTER THREE Total Hydrostatic Force on Surfaces : & HYDRAULIC Conerete Figure 3 - 4: Gravity dam An embankment dam is a gravity dam formed out of loose rock, earth, or a combination of these materials. The upstream and downstream slopes of embankment dams are flatter than those of concrete gravity dams. In essence, they more closely match the natural slope of a pile of rocks or earth Arch dams are concrete or masonry structures that curve upstream into a reservoir, stretching from one wall of a river canyon to the other. This design, based on the same principles as the architectural arch and vault, transfers some water pressure onto the walls of the canyon. Arch dams require a relatively narrow river canyon with solid rock walls capable of withstanding a significant amount of horizontal thrust. These dams do not need to be as massive as gravity dams because the canyon walls carry part of the pressure exerted by the reservoir Figure 3 - 5: Arch dam FLUID MECHANIC: JID MECHANICS CHAPTER THREE Total Hydrostatic Force on Surfaces 83 DRAULICS 4. A buttress dam consists of a wall, or face, supported by several buttresses on the downstream side. The vast majority of buttress dams are made of concrete that is reinforced with steel. Buttresses are typically spaced across the dam site every 6 to 30 m (20 to 100 ft), depending upon the size and design of the dam. Buttress dams are sometimes called hollow dams because the buttresses do not form a solid wall stretching across a river valley. 84 CHAPTER THREE FLUID MECHANICS FLUID MECHANICS CHAPTER THREE 85 Total Hydrostatic Force on Surfaces & HYDRAULICS & HYDRAULICS Total Hydrostatic Force on Surfaces ANALYSIS OF GRAVITY DAM A dam is subjected to hydrostatic forces due to water which is raised on its upstream side. These forces cause the dam to slide horizontally on its foundation and overturn it about its downstream edge or toe. These tendencies are resisted by friction on the base of the dam and gravitational forces which causes a moment opposite to the overturning moment. The dam may also be prevented from sliding by keying its base. A. Vertical forces 1. Weight of the dam Wi=yeVi; We =e Vai Ws = Yc Vs 2. Weight of water in the upstream side (if any) Wi =7Vs 3. Weight or permanent structures on the dam 4. Hydrostatic Uplift Un=y Via th= 7 Vio 7 Upstream Side Downstream Side alle) B. Horizontal Force 2 ri 1. Total Hydrostatic Force acting at the vertical projection of the submerged portion of the dam, Headwater . PHy ha Ls of 2. Wind Pressure ios : the submerged 3. Wave Action face of dam |, 4, Floating Bodies F 5. Earthquake Load III. Solve for the Reaction i A. Vertical Reaction, R, Ry =2F, im Ry = Wh + Wa + W3 + Wa- Ur - Up . Uplift Pressure . B. Horizontal Reaction, R, Diagram Ry = EF; Ry=P IV.Moment about the Toe A. Righting Moment, RM (rotation towards the upstream side) RM = W, x1 + W2 x2 + W3 x3 + Wa x4 R, * Figure 3 - 8: Typical section of a gravity dam showing the possible forces acting B. Overturning Moment, OM (rotation torvards the downstream side) Steps of Solution OM=PytU;, 2+ Ub 2 With reference to Figure 3 - 8, for purposes of illustration, anassumption was made in the shape of the uplift pressure diagram. V. Location of Ry (x) |, Consider 1 unit (1 m) length of dam (perpendicular to the sketch) ll. Determine all the forces acting: FLUID MECHANICS 86 CHAPTER THREE FLUID MECHANICS PORES OF Total Hydrostatic Force on Surfaces & HYDRAULICS & HYDRAULICS Total Hydrostatic Force on Surfaces ee : Ry . 6Rye y = unit weight of water = 9.81 kKN/mi (or 1000 kg/m?) sees B a B? Yc = unit weight of concrete ¥. = 2.4y (usually taken as 23.5 kN/m3) Factors of Safety / \ Factor of safety against sliding, FSs: \a Note: Use (+) to get the stress at point where R, is nearest. In the diagram shown above, use (+) to get gr and (-) to get qu. A negative stress indicates _ compressive stress and a positive stress indicates tensile stress Gince soil cannot carry any tensile stress, the result of Eq, 3 - 14 is invalid if the F i ‘ ae actor of safety against overturning, Fso: stress is positive. This will happen if e > B/6. Should this happen, Eg. 3-415 j _ will be used. - _ RM 1 Fog San Eq.3-13 | 8/2 { e where: = coefficient of friction between the base of the dam and the foundation j | Phen e > B/6 Foundation Pressure x =a/% : | Middle Third ' i = For e < B/6 Bs 1680) Ba ; a=3% From combined axial and bending stress formula: Ry = Ya(a)(qe)(1) Ry = Val3X )qe Po Me q ms T P=R, A=B(1)=B M=Ry,e 3 = 18) 12 Ce] Bf Ry , (Ry (B/2) B ay Be yO CHAPTER THREE 8 8 CHAPTER THREE FLUID MECHANIC! oo Total Hydrostatic Force on Surfaces 89 Total Hydrostatic Force on Surfaces & HYDRAULI HYDRAULICS i, OY. i where: : ne ‘ 1 y = unit weight of the fluid aa Vp = volume displaced. Volume of the body below the liquid surface ARCHIMEDES’ PRINCIPLE 3 A principle discovered by the Greek scientist Archimedes that states that “ To solve problems in buoyancy, identify the forces acting and apply conditions of static body immersed in a fluid is acted upon by an upward force (buoyant force) equal to # weight of the displaced fluid”. This principle, also known as the law of hydrostatics, applies to both floating and submerged bodies, and to all fluids. Consider the body shown in Figure 3 - 9 immersed ina fluid of unit weight The horizontal components of the force acting on the body are all ir equilibrium, since the vertical projection of the body in opposite sides is the same. The upper face of the body is subject to a vertical downward force | Vp = which is equal to the weight of the fluid above it, and the lower face is subject to an upward force equal to the weight of real or imaginary liquid above it, The net upward force acting on the body is the buoyant force. Segoe yy. Peay sp. gr. of liquid Yiiquid If the body of height H has a constant horizontal cross-sectional area such as vertical cylinders, blocks, etc.: Cross-sectional area, A Vo = Vol, — Vol; 4 a sp. gr.of body Be ¥ body 4 Fv Fva BF = Fy, - Fu sp.gr. of liquid Viiguid Figure 3 - 9: Forces acting on a submerged body BE= Fy -Fyi If the body is of uniform vertical cross-sectional area A, the area submerged A, = y(Volz) - y(Vol:) BF = y(Vol - Vol) ‘ S BF =¥ Vp Eq. 3-16 fe sp.gr.of liquid Ytiquid is: CHAPTER THREE 90 FLUID MECHANICS Total Hydrostatic Force on Surfaces & HYDRAULICS STATICAL STABILITY OF FLOATING BODIES A floating body is acted upon by two equal opposing forces. These are, the body’s weight W (acting at its center of gravity) and its buoyant force (acting at the center of buoyancy that is located at the center of gravity of displaced liquid) When these forces are collinear as shown in Figure 3 - 10 (a), it floats in an upright position. However, when the body tilts due to wind or wave action, the center of buoyancy shifts to its new position as shown in Figure 3 - 10 (b) and the two forces, which are no longer collinear, produces a couple equal to W(x). The body will not overturn if this couple makes the body rotate towards its original position as shown in Figure 3 - 10 (b), and will overturn if the situation is as shown in Figure 3 - 10 (c). Che point of intersection between the axis of the body and the line of action of the buoyant force is called the metacenter. The distance from the metacenter (M) to the center of gravity (G) of the body is called the metacentric height (MG). It can be seen that a body is stable if M is above G as shown in Figure 3 - 10 (b), and unstable if M is below G as shown in Figure 3 - 10 (c) If M coincides with G, the body is said to be just stable RM 1 Metacenter 1 iF i Wedge of ; emersion Figure 3 - 10 (a): Upright position Figure 3 - 10 (b): Stable position CHAPTER THREE 9 1 ee Total Hydrostatic Force on Surfaces 4 HYDRAULICS Figure 3 - 10 (c): Unstable position Figure 3 - 10: Forces on a floating body RIGHTING MOMENT AND OVERTURNING MOMENT : RM or OM = W() ae Pee ELEMENTS OF A FLOATING BODY: W = weight of the body BF = buoyant force (always equal to W for a floating body) G = center of gravity of the body Bo'= center of buoyancy in the upright position (centroid of the displaced liquid) By Bo’ = center of buoyancy in the tilted position Vp = volume displaced bei M = metacenter, the point of intersection between the line of action of the buoyant force and the axis of the body c= center of gravity of the wedges (immersion and emersion) s = horizontal distance between the cg’s of the wedges v = volume of the wedge of immersion 6 = angle of tilting MBo = distance from M to Bo GBo = distance from G to Bo MG = metacentric height, distance from M to G FLUID MECHANICS & HYDRAULICS: 92 CHAPTER THREE Total Hydrostatic Force on Surfaces Metacentric height, MG = MB, + GB, Eq. 3 - 21 Use (-) if Gis above Bo Use (+) if G is below Bo Note: M Is always above B, . / VALUE OF MB, "he stability of the body depends on the amount of the righting mom which in turn is dependent on the metacentric height MG. When the body tilts the center of buoyancy shifts to a new position (Bo’). This shifting also causes the wedge v' to shift to a new position v. The moment due to the shifting of the buovant force BF(z) is must equal to moment due to wedge shift F(s) Rolling ae Waterline Section Rolling R THREE CHAPTE 93 PLUID MECHANICS : | Total Hydrostatic Force on Surfaces | & HYDRAULICS Moment due to shifting of BF = moment due to shifting of wedge BF (z) =F (s) BF=y Vp FHyv z= MBo sin® y Vo MBo sin 8 =yvs INITIAL VALUE OF MB, _ For small values of 8, (8 ~ 0 or 0 = 0): ! Volume of wedge, V B/2 poe , me Wedge, volume = v Figure 3 - 11: Rectangular body Consider a body in the shape of a rectangular parallelepiped length L as shown in Figure 3 - 11; Volume of wedge, v = %(B/2)[(B/2) tan 6]L Volume of wedge, v = $LB? tan 8 For small values of 0,5 ~ 2B 94 CHAPTER THREE FLUID MECHANICS Total Hydrostatic Force on Surfaces & HYDRAULICS US. MB, — Vp sind LLB? tanox2B MB, = —————___>_ But for small values of 0, sin 0 = tan 0 Vp sin® 1 3 MB, = a Vp But oo LB? is the moment of inertia of the waterline section, | MBps Eq. 3-23 Note: This formula can be applied to any section. Since the metacentric height MG is dependent with MB,, the stability of a floating body therefore depends on the moment of inertia of the waterline section. It can also be seen that the body is more stable in pitching than in rolling because the moment of inertia in pitching is greater than that in rolling. MOMENT The righting or overturning moment on a floating body is: RM or OM=Wx=W (MG sin6) Eq. 3-24 | FLUID MECHANICS CHAPTER THREE 95 HYDRAULICS Total Hydrostatic Force on Surfaces POR RECTANGULAR SECTION (B/2) sec 8 s/2 ——| Bea] (B/2) tan 6 5/2. ie Centroid of wedge B, (B/2) From Eq. 3 - 22, MB, = vs o> Vasind Vp = BDL where L is the length perpendicular to the figure v = ¥2(B/2){(B/2) tan OJL v= £LB#an 6 Centroid of triangle, ¥ — _ Xp +X +23 From geomet) = —- 3 — _ 0+(B/2)sec@ + (B/2)cos® x= 5 ; 2 8 7-8 + cos0 |= 2 set | 6 \cos6 6 cos0 $i. 2 1B +0820 NS ade | eee 2 6 cos8 1+cos? 0 cos8 w tl | os 96 CHAPTER THREE Total Hydrostatic Force on Surfaces (LB? tan [2 1+ cos? | & HYDRAULI MB, = cosé (BDL)sin® ee sin® i 1+ cos? 6 MB, = —24cos® __cos®__ BDLsin®@ MB, = B? 1+cos* 6 \ 24D cos*@ 2 MB dG 24D \ cos? 6 2 B MB, = ——— 2 MD (sec? 8 + 1) 2 RB . ME, = 2 wT) { (1 + tan? 6) + 1] 2 but sec? 6 = 1 + tan? 6 2 a) B= —-— (2+ tantey= 5 (2, 12(2)D DODO STRESS ON THIN-WALLED PRESSURE VESSELS THIN-WALLED CYLINDRICAL TANK A tank or pipe carrying a fluid or gas under a pressure is subjected to tensile _ which resist bursting, developed across longitudinal and transverse sections pressure p ” FLUID MECHANIC: FLUID MECHANICS CHAPTER THREE 97 & HYDRAULICS Total Hydrostatic Force on Surfaces Consider a pipe of diameter D and thickness ¢ be subjected to a net pressure p. To determine the tangential stress in the pipe wall, let us cut a section of length . s along the diameter. The forces acting on Projection of this section are the total pressure F due to Gunec cise the internal pressure and this is to be F resisted by T which is the total stress of the pipe wall. Applying equilibrium condition; [Fy = 0] F=27 F = pA=pDs °T = St Await T=Sr(sx?) pDs = 2 x [Sr (s x f)] D Tangential stress, Sr = Eq..3- 26 To determine the longitudinal stress, let us cut the cylinder across its length as shown. [2Fu = 0] F=T F=pA Es pe De T= S$, Awan Awa = "Dt T=5, Dt “p EDt= $, Dt D Longitudinal stress, S, = - p = internal pressure — external pressure } CHAPTER THREE 98 Fe eee FLUID MECHANICS. "FLUID MECHANICS CHAPTER THREE 99 In Surfaces & HYDRAULICS & HYDRAULICS Total Hydrostatic Force on Surfaces ee SPHERICAL SHELL : Ifa spherical tank of diameter D and thickness t contains gas under a pressure ; Solved Problems of p, the stress at the wall can be expressed as: : r Problem 3-1 A vertical rectangular plane of height d and base b is submerged in a liquid with its top edge at the liquid surface. Determine the total force F acting on _ one side and its location from the liquid surface. | y J a \ Solution F=yhA Wall stress, S = a Eq. 3-29 | h=d/2 } A=bd F= y(d/2)(bd) F="ybt SPACING OF HOOPS OF A WOOD STAVE PIPE I ee eke y y =h =d/2 3 x abd (bd) /2) e=d/6 # , "Pressure diagram a (triangular prism) Ss Sl yah +e yp = d/2+a/6 Yp = 2d/3 Using the pressure diagram: F = Volume of pressure diagram F=Yalyd)()(0) = ay b # where The location of F is at the centroid of the pressure diagram. S, = allowable tensile stress of the hoo Pp Aj, = cross-sectional area of the hoop Note: For rectangular surface (inclined or vertical) submerged in a fluid with top- edge p = internal pressure in the pipe Wee on the liquid surface, the center of pressure from the bottom is 1/3 of its eight. D = diameter of the pipe CHAPTER THREE 100 FLUID MECHANICS Total Hydrostatic Force on Surfaces & HYDRAULICS Problem 3 - 2 A vertical triangular surface of height d and horizontal base width b is submerged in a liquid with its vertex at the liquid surface. Determine the t6ta force F acting on one side and its location from the liquid surface. Solution F=yhAa h =2d A = Yabd Feyxddx Vobd F=1yba sete Ay y =h =2d/3 58) ve eld (3 bd) (2d /3) e= d/ 12 Pressure diagram ‘ (pyramid) Yoh +e Yp = £d+d/12= 30/4 Using the pressure diagram: F = Volume of pressure diagram F= 4 Abe x height P= £ (bx ydyid)= 4 ybdt Fis located at the centroid of the diagram, which is 4 of the altitude from the base Problem 3 - 3 A vertical circular gate or radius r is submerged in a liquid with its top edged flushed on the liquid surface. Determine the magnitude and location of the total force acting on one side of the gate FLUID MECHANICS & HYDRAULICS CHAPTER THREE 101 Total Hydrostatic Force on Surfaces , Solution F=yhA F=yQ\nr) F=nyr pa oh Ay Ns 4 e= aoe =r/4 (nr )(r) Yport é Yp art rf{4 Pressure diagram Yp = 51/4 (cylindrical wedge) Using the pressure diagram for this case is quiet complicated. With the shape shown, its volume can be computed by integration. Hence, pressure diagram is easy to use only if the area is rectangular, with one side horizontal. Problem 3 - 4 A vertical rectangular gate 1.5 m wide and 3 m high is submerged in water with its top edge2 m below the water surface. Find the total pressure acting on one side of the gate and its location from the bottom. Solution Feyha h =15+2=3.5m F =981(3.5)[(1.5)@)] F=154.51 kN I gate y ole 3 = BOSC) Gee (1.5x 3)(3.5) om y=15-e y=15-0.214 y = 1.286 m CHAPTER THREE FLUID MECHANICS Total Hydrostatic Force on Surfaces | & HYDRAULICS 102 Using the pressure diagram: F= Volume of pressure diagram F= (2271.3) (1.5) F=15.75y F = 15,75(9.81) F= 154.51 kN 3y 2y Pressure diagram (trapezoidal prism) Location of F: Ai = 2y(3) = 6y Az = 'a(37)(3) = 4.57 A=A,+ A =10.5y [Ay = Eay] 10.5y y = 6y(1.5) + 4.5y(1) y = 1.286 m (much complicated to get than using the formula) Problem 3 - 5 A vertical triangular gate with top base horizontal and 1.5 wide is 3 m high. It is submerged in oil having sp. gr. of 0.82 with its top base submerged to a depth of 2m. Determine the magnitude and location of the total hydrostatic pressure acting on one side of the gate. CHAPTER THREE Total Hydrostatic Force on Surfaces UID MECHANICS HYDRAULICS 103 Solution F=yhA h =2+1@) h =3m=y F = [9.81(0.82)](3)[14(1.5)(3) | F=54.3kN ty sey = Ay [(2.5)(3)]() e=0.167m Yah +e Yp = 3.167 m from the oil surface e= Problem 3 - 6 (CE Board May 1994) A vertical rectangular plate is submerged half in oil (sp. gr. = 0.8) and half in water such that its top edge is flushed with the oil surface. What is the ratio of the force exerted by water acting on the lower half to that by oil acting on the upper half? Solution e Force on upper half: Fo = Yo ha Fo= (tw x 0.8)(4/4)[0(d/2)] Fo = 0.1 Yu b d? Force on lower half: Fw = pegr ¥ A Pcg2 = Yo No + Yw Hw Pege = (Yw ¥ 0-8)(d/2) + Yo(d/ 4) Peg2 = 0.65 Yw d Fw= (0.65 yw d){b(4/2)] Fw = 0.325 Yeo b d® F Ratio = —~ Fo 9 2 Ratio = Cowl 308 0.17 ,»bd? 1 04 CHAPTER THREE FLUID MECHANICS Total Hydrostatic Force on Surfaces Problem 3 - 7 (CE Board May 1994) \ A vertical circular gate in a tunnel 8 m in diameter has oil (sp. gr. 0.8) on one. side and air on the other side. If oil is 12 m above the invert and the air pressure is 40 kPa, where will a single support be located (above the inyert of the tunnel) to hold the gate in position? Solution Air; p = 40 kPa Pair Foy = Yoil h A Foun = (9.81 x 0.80)(8) x 4 (8)? hinge Fo) = 3,156 kN I e= — Ay 2g)? = ae =05m $(8)° (8) z=4-e=35m Fair = pair Ar = 40 x (8)? Faip = 2,011 KN The support must be located at point O where the moment due to Fair and Foi is zero. Since Fon > Fair, O must be below Foi). [ZMo = 0] Foa(z - y) = Fair(4 - y) (3,156)(3.5 - y) = 2,011(4 - y) 1.569(3.5 -y) =4-y 5.493 - 1.569y=4-y y=2.62m & HYDRAULICS PLUID MECHANICS & HYDRAULICS CHAPTER THREE 1 05 Total Hydrostatic Force on Surfaces Problem 3 - 8 (CE Board May 1992) A closed cylindrical tank 2 m in diameter and 8 m deep with axis vertical contains 6 m deep of oil (sp. gr. = 0.8). The air above the liquid surface has a pressure of 0.8 kg/cm’. Determine the total normal force in kg acting on the wall at its location from the bottom of the tank. Solution ifr eae Pie = 0.8 kg/cm? yi Pair A Pair = 0.8 kg/cm? = 8,000 kg/m? F, = 8,000(2me 2) = 32,000n kg y=6+1=7m Fy = peg A Peg = (1000 x 0.8)(3) + 8,000 Pex = 10,400 kg/m? Fy = 10,400(2n x 6) = 124,800n kg Solve for e: Fo=yoht A 124,800n = (1000 x 0.8) ht (2n x 6) h=7 =13m va le, HOMO? Ay (2m 6)(13) e = 0.23077 m y2=3-e=2.77m F=F,+ F,=156,800ckg | > Total normal force } } / 106 CARRE FLUID/MECHANICS FLUID MECHANICS CHAPTER THREE 4. (7 Total Hydrostatic Force on Surfaces & HYDRAULICS & HYDRAULICS Total Hydrostatic Force on Surfaces Fy = Fi yi t+ Fe yo Zz Solution (156,800r) y = (32,000n) (7) + (124,800m)(2.77)__ B [2 Mhinge = 0 ] y = 3.63 m > Location of F from the Eom Fz=40(1) Using the pressure diagram: FeyhA=981 h (1)(1.5) =14.715h acto ava F=14,715 io Subs 2m e= —> where y = h Ay oo CLS? P = = —. Forme tee snl (1.5 1)h 12h al eee y i Fy at aveaig eos 12h 800(6) = 4800 8000 Pressure Diagram P; = 8000(8)(2n) = 128,000n kg P2 = ¥4(4,800)(6)(2n) = 28,8007 kg P=P,+ P2=156,800nkg > Total normal force [Py = Pi yn + Po yo] (156,800n) y = (128,000) (4) + (28,8007) (2) y=3.63m ~ Location of P from the bottom Problem 3 - 9 In the figure shown, stop B will break. if the force on it reaches 40 kN. Find the critical water depth. The length of the gate perpendicular to the sketch is 1.5m 14.715 h [os az = 40 12h 0.5h + 0.08333 = 2.718 h =527m=h +0.5=5.77m > critical water depth Problem 3-10 » A vertical circular gate is submerged in a liquid so that its top edge is flushed with the liquid surface. Find the ratio of the total force acting on the lower half to that acting on the upper half. Solution F. Ratio = — 0.5756r f F, Fy £ i a ha A iy Ratio = he yhy Ay F, Ae x = 4r/3x A2 Ratio = 2 hy Ratio = — = 2.475 \ 1 08 CHAPTER THREE \ FLUID MECHANICS FLUID MECHANICS CHAPTER THREE Total Hydrostatic Force on Surfaces & HYDRAULICS 1 09 & HYDRAULICS Total Hydrostatic Force on Surfaces Problem 3 - 11 Solution A 30 m long dam retains 9 m of water as shown in the figure. Find F=yhA the total resultant force acting on the ; dam and the location of the center of be oey ae 5 pressure from the bottom. h = 4.167 1m =n A= %2(1)(2.61) A= 1.305 m? F = (9810 x 0.83)(4.167)(1.305) F=44,277N 2m Solution F = 44.277 kN | F=yha F=9,81(4.5)[(30)(10.392)] F= 13,763 kN Problem 3 - 13 I, An inclined, circular oe ae gate with water on one a side is shown in the 75 (30) (10.392)° figure. Determine the (30 x 10.392)(4.5 / sin 60°) total resultant force e=1.732m acting on the gate. y = ¥(10.392) - 1.732 y=3464m or i 4 (10.392) = 3.464 m Problem 3 - 12 i Solution The isosceles triangle gate shown a in the figure is hinged at A and a Ms weighs 1500 N. What is the total he 0S sin bv hydrostatic force acting on one side h =2.433 of the gate in kiloNewton? F = 9,81(2.433) # (1)? F =18.746 kN Oil (S = 0.83) | FLUID MECHANICS \ & HYDRAULICS _ FLUID MECHANICS & HYDRAULICS 11 0 CHAPTER THREE Total Hydrostatic Force on Surfaces CHAPTER THREE 111 Total Hydrostatic Force on Surfaces Problem 3 - 14 Ie 4(1.5)(8.6)° ; 7 : : ; \, j ()) @= == = =. eee The gate in the figure shown is 1.5 m wide, hinged at point A, and rests Ay (1.5x3.6)(7.21) against a smooth wall at B. Compute (a) the total force on the gate due to seawater, (b) the reaction at B, and (c) the reaction at hinge A. Neglect the weight of the gate. } e=0.15m x=1.8- 0.15 x=1.65m [=Ma = 0] F(x) - Ra(2) = 90 218.25(1.65) = 2 Re Rz = 180 kN Seawater s= 1.03 ‘ [ZFu = 0] Ran + F sin 9 -R,=0 Ran = 180 - 218.25 sin 33.69° Ray = 58.94 kN [= F, = 0] Wall ae Ray - P.cos 6 = 0 | Ray = 218.25 cos 33.69° Ray = 181.6 kN Ra= JRay2 +Rau? = ¥(181.6) + 68.94)" ; Ra = 190.9 kN Solution ‘ ¥ Problem 3 - 15 = 324 22 Determine the magnitude us ‘. ie and location of the total ‘Cox hydrostatic force acting on tan 0 = 2/3 5m the 2m x 4 m gate shown 6 = 331692 in the figure. h Glycerin, s = 1.26 = L 7a ‘ sin® a 4 iy = ————_ sin 33.69° y =7,.21m (a) Pry ha F = (9.81 x 1.03)(4)[(1.5)(3.6)] F = 218,25 kN 1 1 2 CHAPTER THREE | Total Hydrostatic Force on Surfaces \ Solution P= peg A Pog = =yh + p Peg = (9.81%1.26)(3) + (9.81)(1.5) + (9.810,80)(1) + 32 Peg = 91.645 kPa F =91.645(2 x 4) F = 733.16 kN _ Solving for e: Solve for h and 7: Fey ha 733.16 = (9.81x1,26) h (2 x 4) =7414m ¥ = h /sin 60° = 7.414 / sin 60° =8.561m y ole seer Ay (2x 4)(8.561) e=0.156m > Ss z=2-¢e=1844m Therefore, F is located 1.844 from the bottom of the gate. FLUID MECHANICS. & HYDRAULICS 15 m by 0.5 m shown in the Solution FLUID MECHANICS CHAPTER THREE 113 & HYDRAULICS Total Hydrostatic Force on Surfaces Problem 3 - 16 (CE November 1997) Determine the magnitude of the force on the inclined gate figure 001. The tank of water is completely closed a and the pressure gage at the Figures bottom of the tank reads 90,000 N/m. Use 9,800 N/cu. m. for water. os) F=pgA P2- Peg = Yo 90000 ~ prey = 9,800(2.65) Peg = 64030 Pa F = 64030 (0.5 x 1.5) F = 48,022.5 N P2 = 90,000 Pa Problem 3 - 17 The gate shown in the figure is hinged at A and rests on a smooth floor at B. The gate is 3 m square and oil of having sp. gr. of 0.82 stands to a height of 1.5 m above the hinge A. The air above the oil surface is under a pressure of 7 kPa above atmosphere. If the gate weighs 5 kN, determine the vertical force F required to open it. / f fi 114 CHAPTER THREE | Total Hydrostatic Force on Surfaces | \ CHAPTER THREE 115 FLUID MECHANICS Total Hydrostatic Force on Surfaces & HYDRAULICS FLUID MECHANIC & HYDRAULIC ‘Problem 3 - 18 (CE Board) Iron pins 20 mm in diameter are used for supporting flashboards at the crest Tests show that the yield point of iron to be 310 MPa ting the dynamic effect of water on flashboards what is the proper spacing, 5S, of the iron pins, d when water flows 150 mm deep of masonry dams. (extreme fiber stress). Neglec and assuming static conditions, so that the flashboards 600 mm high will yiel over the top of the flashboards. Solution 0.15 m = N Solution P= peg A Peg = Pair + Walt Peg = 7 + 9.81(0.82) (2,56) Pg = 27.59 kPa P = 27.59 [(3)()] P = 248.34 kN P=yhA 248.34 = (9.81%0.82) ht (3 x 3) h =3.43m ic Flashboard j= Hee 73H8 _ sin 45° sin 45° y =485m 3 sin 45° = 212m Moment capacity of one iron pin (20 mm 2): I 1 3 [Fi Ki Mc/T] ju oO. M(2) Ay (3«3)(4.85) S10 eg (20) e= 0.155 m : £15 +e M = 243,473.43 N-mm x= 1.655 m M = 0.24347 kN-m [2Ma =0] ) Moment caused by F (considering 5 m width of flashboard): P(x) + W(1.06) - F(2.12) = 0 Mr=Fxy 2.12F = 248.34(1.655) + 5(1.06) ; F=yhA whereA=065 , : F = 9.81(0.45)[0.6 5] F=196.37 KN 17 6 CHAPTER THREE Total Hydrostatic Force on Surfaces F= 2.6495. y=0g-e Ee 5(S)(0.6)? scenes tr ape MUO) Ay (0.6)(0.45) e = 0.067 m y = 0.3 - 0.067 = 0.233 m Mr=Fxy=M 2.649 S x.0.233 = 0.24347 S = 0.394 m = 394 mm Problem 3 - 19 The semi-circular gate shown in Figure 28 is hinged at B. Determine the force F required to hold the gate in position. Solution h = 7 =10-1.698 h = 7 =8302 tt P=yhA P = 62.4(8.302)[4/2n(4)?] P= 13,019.89 lbs I eH 22 A Ty = 0.1098 4 fi I, = 0.1098(4)4 Ty = 28.11 ft! 28.11 4 ft P “i 4n(4)? (8.302) ues e = 0.1347 ft b = 1,698 - 0.1347 = 1.5633 ft [=Mz = 0] P(b) = F(4) 13,019.89(1.5633) = F(4) F = 5088.5 Ibs CHAPTER THREE 1 1 7 Total Hydrostatic Force on Surfaces FLUID MECHANICS FLUID MECHANICS & HYDRAULICS & HYDRAULIC Problem 3 - 20 At 20 °C, gage A in the figure reads 290 kPa absolute. The tank is 2 m wide perpendicular to the figure, Assume atmospheric pressure to be 1 bar. Sp. gr. of mercury = 13.6. Determine the total pressure acting on side CD. Solution Solving for h: pA =Lyh + Prop 290 = (9.81 x 13.6)(0.70) + (9.81)h + 175 h=2.2m * Total force on side CD; (Note: 1 bar = 100 kPa) Pi pi = 175 - 100 teat bi p=75 kPa pr = 9.81(2.9) p2 = 28.449 kPa 7 F: = pi(3.9)(2) ae F; = 75(3.9)(2) PF, = 585 kN Fa = Ya pr(2.9)(2) F> = ¥2(28.449) (2.9) (2) F. = 82.5 kN F=F,+F, F = 667.5 kN 3.9m 2.9m Po Py 2m FLUID MECHANICS CHAPTER THREE / 118 & HYDRAULICS Total Hydrostatic Force on Surfaces Problem 3 - 21 \ The funnel shown in the figure is full of water. The volume of the upper part is 90 liters and the lower part is 74 liters. What is the force tending to push the plug out? 1.4m Area = 100 cm? 16m Plug: Area = 460 cm? Solution Since the plug area in contact with water is horizontal, the pressure all over itis uniform. The shape of the container does not affect the pressure on the’ plug. _ Force = pxA Force = 9,810(3)(— ) Force = 1353.78 N Problem 3 - 22 In the figure shown, the gate AB rotates about an axis through B. The gate width is 1.2 meters. A torque T is applied to the shaft through B. Determine the torque T to keep the gate closed. Solution REE CHAPTER TH 1 1 9 FLUID MECHANICS Total Hydrostatic Force on Surfaces HYDRAULICS F= yh A P= 9.81(2.6)(1.6 x 1.2) F = 48.97 kN ee ot AY 1.2(1.6)° ee (1.6 x 1.2)(2.6) e=0.082m z=08-e z=0.718 m T=Fxz T = 48,97 x 0.718 T = 35.16 kN-m Problem 3 - 23 (CE Board) A cubical box, 1,5 m on each edge, has its base horizontal and is half-filled with water. The remainder of the box is filled with air under a gage pressure of 82 kPa. One of he vertical sides is hinged at the top and is free to swing inward. To whatélepth can the top of this box be submerged in’an open body of fresh water without allowing any water to enter? Solution 9.81(0.75) 82 kPa = 7,36 kPa CHAPTER THREE 120 Total Hydrostatic Force on Surfaces [E Muinge = 0] Fs (x) - Fy (0.75) - Fy (1.25) =0 i= PairA F, = 82{(1.5)(1.5)] = 184.5 kN Fy = ¥4(7.36)(0.75) (1.5) F,=4.14 kN F3=yhA Fy = 9.81h [(1.5)(1.5)] Py = 22.07h x=O0.75 +e 155)? pyeks Sie ee Ay — [(1.5)(1.5))h = 0.1875. h x= 075 + 01875 Jn Equation (1): 0.1875 22.07 h (0.75 + = ) 184.5(0.75) - 4.14(1.25) = 0 t 16.55h + 4,138 - 138.375 - 5.175 = 0 16.55 h = 139.412 h =8.42m h=h -0.75 h=7.67m FLUID MECHANI FLUID MECHANICS CHAPTER THREE 1 2 1 & HYDRAULICS Total Hydrostatic Force on Surfaces Problem 3 - 24 Whe Find the magnitude and location of ps ay the force exerted by water on one side of the vertical annular disk shown. 4m LG R=1.5m Solution F=yhA F=9,81(4)[n(1.5)2- x(1)] F=154.1 kN Location of F: I £1.5)4 - £1)? oo = atid BY? AZ T/A). Ay — nf(.5)° -(1)"](4) e=0.203m Yp =4+ 0,203 = 4.203 m below the w.s. Problem 3 - 25 The gate in the figure shown weighs 5 kN for each meter normal to the paper. Its center of gravity is 0.5 m from the left face and 0.6 m above the lower face. Find h for the gate just to come up to the vertical position. 122 CHAPTER THREE : f FLUID MECHANICS Total Hydrostatic Force on Surfaces i & HYDRAULICS Solution Considering 1m oar Fy = ¥2 (9,.81h)(h)(1 F, = 4,905 h? kN Fy = 9,81h(1.5)(1) F)=14.715h kN [ZMo = 0] F\(h/3) + W (0.6) - F2(1.5/2) = 0 4,905]? (h/3) + 5(0.6) - 14.715h (0.75) =0 1.635/8 - 11.04h + 3 =0 Solve It by trial and error ht = 0.2748 m FLUID MECHANICS & HYDRAULICS Total Hydrostatic Force on Surfaces Problem 3 - 26 In Problem 3 - 25, find h when the force against the “stop” is a maximum. Solution [ZMo = 0] F,(u/3) + W (0.6) + P(1.5)- P2(1.5/2) = 4.90517 (1/3) + 5(0.6) + P(1.5) - 14.715) (0.75) = P = 1.0918 - 7.358h + 3 ae = 3.27 I? - 7.358 =0 I= 2.25 h=15m Problem 3 - 27 Determine the force due to water ¥ acting on one side and its location on | the parabolic gate shown using integration. | 4m APTER THREE re 123 Solution dF =pdA p-yy dA = 2x dy By squared property of parabola: dF = yy (2 (2./y/3) dy dF = 2.31yy?/? dy F 3 fer = 2oly fren 0 0 5 3 F=231y BY = 2.31(9.81) 2 [ 3°/2 - 05/2] 0 F=141.3 kN 4 Location: Pyp= foe 3 141.3 yp = i 3 31yy9/2dy) 0 Yp = 0.1604 frre 0 7/2 e yp = 0.1604 [e [ry?! i Yp = 0.1604 (2/7) [ 37/2 - 07/2] yp = 2.14 m below the w.s. CHAPTER THREE 124 Problem 3 - 28 In the figure shown, find the width b of the concrete dam necessary to prevent the dam from sliding. The specific gravity of concrete is 24 and the coefficient of friction between the base of the dam and_ the foundation is 0.4. Use 1.5 as the factor of safety against sliding. Is the dam also safe from overturning ? Solution Consider 1 m length of dam W.= Ye Ve W.= ¥2.4)[O)(6)(D)] W. = 14.4 by PHy hA F=y(2.25)[(4.5)(1)] F =10.125y Ry = F = 10.1257 Ry=W. Ry= 14.4 by Ry _ 0.4(14.4by) ~~ 40.125y b= 2.637 m 1s _ RM OM W, (0/2) F(L5) FS, ES,= FS) = 10.125y(1.5) 14.4(2.637)y(2.637 / 2) / Total Hydrostatic Force on Surfaces / FLUID MECHANICS & HYDRAULIC i =3,3>1 (Safe) CHAPTER THREE Total Hydrostatic Force on Surfaces FLUID MECHANICS & HYDRAULICS 125 Problem 3 - 29 (CE Board) 4 dam is triangular in cross-section with the upstream face vertical. Water is flushed with the top. The dam is 8 m high and 6 m wide at the base and Weighs 2.4 tons per cubic meter. The coefficient of friction between the base and the foundation is 0.8. Determine (a) the maximum and minimum unit pressure on the foundation, and the (b) factors of safety against overturning and against sliding. Solution Sp. gr. of conc, Scone = Teone. Yw 2.4 x 1000 Sp. gr. of conc, Scone = ——————_ = 2.4 ae ; 1000 Consider 1 m length of dam W=7.V = (24) -(6)(8)(0)] W=57.6y where y = unit wt. of water F=yhA = y(4)(8 x 1) F=32y * Ry = P = 32y R, = W=57.6y RM = W(4) = 57.6y(4) RM = 230.4y OM = P(8/3) = 32y(8/3) OM = 85.33y = . RM-OM Ry <—X— . R z = 23047-85337 5 19m soil pressure at the toe > soil pressure at the heel _ wR, _ 0,8(57.6y) Re 32y FS, = 1.44 nee RM _ 230.4y ~~ OM 85.334 Fe Problem 3 - 30 (CE Board May 1992) A gravity dam of trapezoidal cross-section with one face vertical and horizontal base is 22 m high and has a thickness of 4 m at the top. Water upstream stands 2 m below the crest of the dam masonry is 2.4 A Neglecting hydrostatic uplift: 1 Find the base width B of the dam so that the resultant force will cut the extreme edge of the middle third near the toe. 2 Compute the factors of safety against sliding and overturning Use tt =10:5) B Considering uplift pressure to vary uniformly from full hydrostatic pressure at the heel to zero at the toe: 1 Find the base width B of the dam so that the resultant force will act a the extremity of the middle third near the toe The specific gravity of 2 Compute the maximum and minimum compressive stresses acting. against the base of the dam FLUID MECHANICS CHAPTER THREE 127 & HYDRAULICS Total Hydrostatic Force on Surfaces Figure: Solution A. Neglecting hydrostatic uplift: I. Consider 1 m length of dam Il. Forces Wi = yeV1 = (y x 2.4)[(4)(22)(1)] Wi 201.27 Wa= (yx 24)[ % (B-4)(20)(1)] W) = 24By - 96y f F=yh A =y (10)[(20)(1)) F = 200w * {Il. Reaction Ry = SF, = P Ry = 200y Ry =5Fy=Wi+ Wp = 211.2y + 24By - 96y R, = 24By + 115.2y CHAPTER THREE Total Hydrostatic Force on Surfaces 128 4m pressure diagram 28/3 —___,| IV Moment about the toe Fee ea lear! RM = W,(B - 2) + Wa[2 (B - 4)| = 211.2y(B - 2) + (24By - 96y) [4 (B-4)] = 211.2By - 422.4y + 16B2y - 128By + 256y RM = 16By + 83.2By - 166.4y OM = F(20/3) = 200(20/3) ‘ OM = 1333.33y Vv Location of R R, ¥ =RM-OM Since the resultant torce will pass through the extreme edge of the middle thirds near the toe, ¥ = B/3. Then, 8B?y + 38.4By = 16B2y + 83 2By - 1499 73y 8B? + 44.8B - 1499 73 = 0 /FLUID MECHANICS f & HYDRAULICS CHAPTER THREE FLUID MECHANICS ; Total Hydrostatic Force on Surfaces & HYDRAULICS _ 44.84 (448)? 4(8)(-1499.73) is 2(8) 129 B=11.175 m Factors of Safety: Factor of safety against sliding: BG ee R, — (0.5)[24(11.175)y + 115.27] 200, FS, = 0.9585 Factor of safety against overturning: 73,= 8M. OM 16(11.175)* y + 83.2(11.175)y — 166.4y " 1333.33y FS, = 2.07 ; A. Considering hydrostatic uplift: Uplift forcé, LI = 2 (20/)(B)(1) = 10Bw R,y=W,+Wo-U = 24By + 115.2y = 10By R, = 14By + 115.2y RM = W,(B - 2) + Wal 2 (B - 4)] RM = 16B2y + 83.2By - 166.4y OM = F(20/3) + U(2B/3) = 200)(20/3) + 10By (2B/3) OM = 6.67B + 1333.33y R,¥ =RM-OM (14By + 115.2y)(B/3) = 16B2y + 83.2By - 166.4y - (6.67B2y + 1333.33y) 4.66B2 + 44.8B - 1499.73 = 0 pa = aa84 (44.8)? — 4(4.66)(-1499,73) 2(4.66) B=13.766m CHAPTER THREE 130 Total Hydrostatic Force on Surfaces Foundation stress: ¥ = B/3 ¥ = 13.766/3 = 4.59 m e=B/2- X¥ =2,2943 m R i= Elie B B Ry = 14(13.766)(9.81) + 115.2(9.81) Ry = 3020.73 kN ee il : ee 13.766 13.766 yr = - 438.87 kPa qu = 0 kPa | FLUID MECHANIC ' & HYDRAULIC plution y= 3 (6)=2m Problem 3 - 31 (CE Board May 2002) The section of a concrete gravity dam shown in the figure. The depth of water at the upstream side is 6m. Neglect hydrostatic uplift and use unit weight of concrete equal to 23.5 kN/m3. Coefficient of friction between the base of the dam and the foundation is 0.6. Determine the following: (a) factor of safety against sliding, (b) the factor of safety against overturning, and (c) the overturning moment acting against the dam in kN-m CHAPTER THREE 1 3 1 q Se ecuuies: Total Hydrostatic Force on Surfaces P= Yoh A = 9.81(3)(6 x 1) PF = 176.58 kN W, = Ye Vi = 23.5[2(8)(1)] W, = 376 kN Wr = V2 = 23.5[12(2)(8)(1)] W) = 188 kN x, =4-12(2) =3 xy = (2/3)(2) = 1.333 m R, = F = 176.58 kN Ry = W, + Wa = 376 + 188 R, = 564 kN HR, eee 3 Rate 0.6(564) = ’ =1.916 FS.= “776.58 RM = Wixi + Wp x = 376(3) + 188(1.333) RM = 1378.604 kN-m OM=Fxy = 176,58(2) OM = 353.16 kN-m ~ overturning moment RM Ga F OM FS; = 1378.604 _ 3.904 353.16 134 CHAPTER THREE Total Hydrostatic Force on Surfaces | Consider 1 m length of dam ee : ae Il Forces Wr=¥% Vi W, = 23.54 [¥2(5.2)(52)(1)} = 3,183 kN Ws = 23.54 [(7)(52)(1) = 8,569 kN Wa = 23.54 [¥2(26)(52)(1)] = 15,913 kN Wa = 9.81[4(5)(50)(1)) = 1,226.3 kN U = ¥3(490.5)(23.2)(1) = 5,690 kN F=yh A=9.81 (25)[50(1)] = 12,263 kN Ill Reaction Ry = F = 12263 kN Ry = Wy + Wo + Ws + Ws - U = 3,183 + 8,569 + 15,913 + 1,226.3 - 5,690 R, = 23,201.3 kN IV. Moment RM = Wi(34.73) + W2(29.5) + W3(17.33) + W4(36.53) = 3,183(34.73) + 8,569(29.5) + 15,913(17.33) + 1,226.3(36.53) RM = 683,900.12 kN-m OM = F(50/3) + LI(30.47) = 12,263(50/3) + 5,690(30.47) OM = 377,758 kN-m Vv Location of R, R, ¥ = RM-OM 23,201.3 x = 683,900.12 - 377,758 ¥ =138.2m (a) The resultant force 1s 13.2 m from the toe ay UR ' R, 0.75(23,201.3) FS, = ———_—. = 1.42 12,263 FLUID MECHNICS- & HYDRAILICS: FLUID MECHANICS & HYDRAULICS CHAPTER THREE 1 3 5 Total Hydrostatic Force on Surfaces by rg, 2 BM 683,900.12 “OM 377,758 FS, = 1.81 (d) Foundation pressure e=B/2-%X e = 38.2/2-13.2=5.9m < B/6 a=-H{14%] B By 38.2 38) Stress at the toe, (use “+”); qr =-1,170.21 kPa Stress at the heel, (use “-”) gn = -44.52 kPa (e) Unit horizontal shearing stress, S, S.= Ry _ 12,268 = 321 kPa AWtase 38.2(1) Problem 3 - 34 The submerged curve AB is one quarter of a circle of radius 21m and is located on the lower corner of a tank as shown. The length of the tank perpendicular to the sketch is 4m. Find the magnitude and location of the horizontal and vertical components of the total force acting on AB. 1 3 6 CHAPTER THREE Total Hydrostatic Force on Surfaces Solution Ey = yh A Fu = 9.81(5)[(4)(2)] Fu = 392.4 kN y=1lt+e I 2 es Eh h=Y=5m Ay po AO) 2 [(4)(2)]6) ll ¢ = 0.067 m y=1+ 0.067 € 20 y = 1.067 m Fu Therefore; Fy is acting 1.067 m below B Fy = Weight asce Fv = yVascp Vasco = 4(A) A=Ai\+A. Ay = (4)(2) = 8 m? Ar = Ya m(2)? = 3.14 m? A=8+3.14 A =11.14 m? Vasco = 4(11.14) = 44.56 m3 Fy = 9.81(44.56) Fy = 437.13 kN Location if Fy AX =Ai x + Azx y= 1m we at. AQ) 3n 3n t = 0.849 m 11.14 ¥ = 8(1) +3.14(0.849) ¥ =0.957m Therefore; Fy is acting 0.957 to the right of A FLUID MECHANIC & HYDRAULIC shall also pass through the center of the FLUID MECHANICS CHAPTER THREE 1 37 & HYDRAULICS Total Hydrostatic Force on Surfaces Another way of solving x: Since unit pressure is always normal to the surface and a normal to the circle passes through its center, then the total force F circle O, hence the moment about O due to F or due to Fy and Fy is zero. [EMo = 0] Fy x- Fury =0 437.13 ¥ =392.4(1.067) x = 0.9578 m Note: This is true to all cylindrical or spherical surfaces. Problem 3+ 35 (CE Board) The crest gate shown consists of a cylindrical surface of which AB is the base supported by a structural frame hinged at O. The length of the gate is 10 m. Confpute the magnitude and location of the horizontal and vertical components of the total pressure on AB. 6 10cos60° = 5m Solution 10 sin 60° = 8.66 m 1 33 CHAPTER THREE Total Hydrostatic Force on Surfaces Fo=yha ae Fu = 9.81(4.33)[10(8.66)] Fu = 3679 kN y = 4 (8.66) = 2.887 m Therefore; Fy is acting 2.887 m above O Fy =y Vasc Vasc = Vaosc - Vaon Vasc = $39 (8.66) x 10 - (10)? [60°45] x 10 Vasc = 125.9 m3 Fy = 9.81(125.90) Fy = 1235 kN Moment about O due to Fy and Fy = 0 Py (x) = Fu (y) 1235 x = 3679(2.877) x=8.57 m Therefore; Fy is acting 8.57 m to from O Problem 3 - 36 (CE May 1999) Calculate the magnitude of the resultant force per meter length due to water acting on the radial tainter gate shown in Figure 021. Figure 021 FLUID MECHANICS & HYDRAULICS Solution CHAPTER THREE 1 39 : ICS eID MECHA Total Hydrostatic Force on Surfaces & HYDRAULICS Considering 1 meter length: Fy=yhaA Fu = 9.81(3)(6 x 1) Fu = 176.58 KN Fy=y7Vs As = Asector ~ Atriangle 1(6)"(60) 462 sin 60° = tt -14(6)2 sin 60 = 360° 6) Ag = 3.26 m? Fy = 9.81(3.26 x 1) Fy = 31.98 KN Fe Py? +P? F= (176.58)? + (31.98)? F=179.45 KN Problem 3 - 37 Calculate the magnitude of the 5 resultant pressure on a 1-ft-wide strip of a semicirculaf taintor gate shown in Figure-12. Solution Figure-12 Fy = Peg A Fy = (62.4 x 2.5)(5 x 1) = 780 Ibs Fy =y Vasc Fy = 62.4 x [ £ 6)2(1)] = 1225 Ibs F= (Fu) +) F = 4 (780)? + (1225)? = 1452 Ibs 1 40 CHAPTER THREE Total Hydrostatic Force on Surfaces Problem 3 - 38 Determine the magnitude of the horizontal and_ vertical components of the total force per meter length acting on the three-quarter cylinder gate shown Solution a. FLUID MECHANICS & HYDRAULICS Fu=y it A Fy = 9.81(3){(1)(2)] Fy = 58.86 kN Fy = yVol Fy = 9.81[4(2)(1) + 0.75[x(2)2(1)] Fy = 170.94 kN FLUID MECHANICS CHAPTER THREE & HYDRAULICS Total Hydrostatic Force on Surfaces Problem 3- 39 (CE Board November 1993) In the figure shown, the 1.20 m diameter cylinder, 1.20 m long is acted upon by water on the left and oil having sp. gr. of 0.80 on the right. Determine the components ‘of the reaction at B if the cylinder weighs 19.62 KN. Solution Fin=yhaA Fin = 9.81(1.2)(1.2 1.2) Fin = 16.95 kN Fy =yV Fy = 9.81[%4 m (0.6)2(1.2)] Fy = 6.657 kN Fusyh A= (9.81 x 0.8)(0.6)(1.2 « 1.2) Fin = 6,78 kN Fy = V2 = (9981 x 0.8) [4 x (0.6)2(1.2)] Fy, = 5.32 kN [2Fu = 0] Fin - Fi - Ran = 0 Rex = 16.95 - 6.78 Rey = 10.17 kN [=Fy = 0] Ray + Fy + Fy.- W=0 Rey = 19.62 - 6.657 - 5.32 Ray = 7.64 kN 141 142). CU Ree FLUID MECHANICS FLUID MECHANICS Bee TETHREE)) aaa Total Hydrostatic Force on Surfaces & HYDRAULICS -& HYDRAULICS. Total Hydrostatic Force on Surfaces Problem 3 - 40 F, = 0.0505 kN F, = Fy - Fy F, = 0.114 - 0.0505 Fy = 0.0635 kN = 63.5 N downward Fp = (9.81 x 0.82) (0.00628) An inverted conical plug 400 mm diameter and 300 mm long closes a 200 diameter circular hole at the bottom of a tank containing 600 mm of oil havi sp. gr. of 0.82. Determine the total vertical force acting on the plug. Solution _ Problem 3 - 41 E sions A 2 m diameter horizontal cylinder 2 m long plugs a Im by 2m rectangular 0.6m eee eae ‘aie hole at the bottom of a tank. With what force is the cylinder pressed against the bottom of the tank due to the 4-m depth of water? ey 0.15 Mm jj Solution 0.15 m hy = 2.x (1 egs 30°) hy = 1,732 m ( Io =4- hy °4ott hy = 2.268 m Fr=yVi EY Vi = Ai “x2 015m) Area, A; = Area of rectangle DEFG - Ay 1)7 (60° es Area of segment, Ag = aes - ¥2(1)(1) sin 60 Fi=yVi F, = (9.81 x 0.82)[n(0.1)2(0.45)] F, = 0.114 kN Area of segment, Aq = 0.09059 m? Area, A; = 1(2.268) - 0.09059 Area, A; = 2.1774 m? Reus Vi = 2.1774(2) = 4.355 mi Vi= VingewaVew . F; = 9.81(4.355) 2 Frustum Cylinder e n(0.15)] 5 F, = 42.72 kN ys Oy ol %= (0.2)? + 0.2)(0.1) + (0.1)? - n(0.1)2(0.15) fe ai eas V2 = 0.00628 m3 Vo =Arx2 144 Ce eReE FLUID MECHANICS FLUID MECHANICS COAPTERTUREE | one Total Hydrostatic Force on Surfaces & HYDRAULICS & HYDRAULICS Total Hydrostatic Force on Surfaces _. m(1)? (120°) Byte: Fu=yha Aree ot seement, A= arm a eaenict) #21 120 Fu = 9.81(6.12){(4.24)(1] Area of segment, Az = 0.614 m2 Fy = 254.56 kN V2 = 0.614(2) = 1.228 m3 F, = F3 = 9.81(1.228) Fy=y Venadea F, = F3 = 12.05 kN i Vehaded = (Asemicircle + Atrapezoid) X 1 Nebe peehees: ; Vshaded = 4 (3)? a 48.28 (4.24)| (1) et force = F; - F, - Fs | Vsnadea = 40.1 m3 Net force = 42.72 ~ 12.05 - 12.05 ETON Net force = 18,62 kN Fy =9,81(40.1) Fy = 393.38 kN Problem 3 - 42 Problem 3 - 43 The gate shown is a quarter circle 25 m wide. Find the force F just sufficient to prevent rotation about hinge B.° Neglect the weight of the gate. In the figure shown, determine the horizontal and vertical components of the total force acting on the cylinder per m of its length. Solution Solution Fy = yh A Fy = 9.81(1)(2.5 x 2) Fy = 49.05 kN im Net Vertical Projection Fy=y Vase 2.5m Fy=9.81[(2 x 2) - 0. ancl 5) Fy = 21.05 kN CHAPTER THREE Total Hydrostatic Force on Surfaces FLUID MECHANICS & HYDRAULICS 146 Solve for z and x : Since the surface 1s circular, Mo = 0. due to Fr, and Fy Fy (2) = Fu (2/3) 21.05(z) = 49.05(2/3) z= 1.55 m0 y=2-zZ= x=0.45m [2Mz = 0] Fis (2/3) + Ev (x) - F(2) =0 2F = 49,05(2/3) + 21.05(0.45) Olw 2 ao 188 t F= 21.09 kN Problem 3 - 44 he cylindrical tank shown has a hemispherical end cap 4m Compute the horizontal and vertical components of the total force due to oil and water acting ? ‘ Oil, s = 0,80 on the hemisphere ' 4 aye ee. Solution 4(3)/3x = 1.273 4(3)/3n = 1.273 FLUID MECHANICS & HYDRAULICS ~ Forces due to oil: Fuo = Pego A Fuo = (9.81 x 0.80)(7 - 1.273) x Ven(3)? Fyo = 635.4 kN Fyo = Yo Vo V, = Volume of imaginary oil above the surface V. = Volume of half'cylinder - Volume of ¥ sphere Vi, = VYer(3)2(7) - 44 4 (3)? Vo = 70.686 m? Fyo = (9,81 x 0.80) (70.686) Fvo = 554.74 kN Forces due to water: Fuw = pegew A Faw = [(9.81 x 0.8)(7) + 9.81(1.273)] x Yen(3)? Fuw = 953.19 kN Fyw= Weight of real and imaginary oil above the surface + weight of real water above the surface Fyw= (9.81 x 0.8)x Yan(3)2(7) + 9.81 x %4 4 2(3)3 Fyw= 1,054.01 . Total horizontal force, Fy = Fao + Faw Total horizontal force, Fr = 635.4 + 953,19 Total horizontal force, Fy = 1,588.59 kN > Total vertical force, Fy = Fyw - Fvo Total vertical force, Fy = 1,054.01 - 554.74 Total vertical force, Fy = 499.27 kN Another way to solve for the total vertical force, Fy: Fy = weight of fluid within the hemisphere Fv=Yo Vot Yw Vw Fy = (9.81x0.8)[ 4 4 (3)3)] + 9.81[ tx $x (3)3)] Fy = 499,27 kN ; CHAPTER THREE Total Hydrostatic Force on Surfaces pee nanan cee EEE EEEEEEDEEEEEEE neeeeaeed 1 48 CHAPTER THREE FLUID MECHANICS _ FLUID MECHANICS CHAPTER THREE 1 49 Total Hydrostatic Force on Surfaces & HYDRAULICS & HYDRAULICS Total Hydrostatic Force on Surfaces Problem 3 - 45 Problem 3 - 46 Determine the force required to open the quarter-cylinder gate shown The weight of the gate is 50 kN acting 1.2 m to the right of O Pressurized water fills the tank shown in the figure Compute the nel hydrostatic force acting on the hemispherical surface Hemispherical Hemispherical surface . surface ‘ 5.5m Solution Convert 100 kPa to its equivalent pressure head, he, h Solution Heg = PB Since the gate has circular surface, Y the total water pressure passes _ 100 ea through point O which is also the ~ 981 location of the hinge, therefore the Neg = 10.194 m ein moment due to water pressure about the hinge is zero. h = 10.194 - 5 h=5.194 m [EMo= 0) F(2.5) = 50(1.2) + Fr(0) 100 kPa F=24kN F = Weight of imaginary water above the hemispherical surface Fava Vs Problem 3 - 47 V» = Volume of cylinder + Volume of hemisphere A hemispherical dome shown is filled with oil (s = 0.9) and is attached to the Viv = (2)?(5.194) + x 4 x (2)* floor by eight diametrically opposed bolts. What force in each bolt is required Va = 82.025 m3 i : to hold the dome down, if the dome weighs 50 kN? F = 9.81(82.025) F = 804.7 kN 150 CHAPTER THREE FLUID MECHANICS ; FLUID MECHANICS CHAPTER THREE 1 5 1 Total Hydrostatic Force on Surfaces & HYDRAULICS _ & HYDRAULICS Total Hydrostatic Force on Surfaces P Solution {2 Fy = 0) F + Far = Eou = 0 P= Foa- Fan Foit = Y Vou above the cone Foi = (9.81x0.8) [7(0.805)2(5) - 4-n(0.805)°(3)] Est = 63.91 kN Fair = Pair A Pyir = 20 [ $ (1.61)? ] = 40.72 KN F = 63.91 - 40.72 F =23.19kN Solution Fy =) Virasiniry oilaboxe thadome Fy = (9.81x0.9)[n(2)? (8) - 22(2)3] Problem 3 - 49 Fy = 739.66 kN A 300 mm diameter steel pipe 12 mm thick carries water under a head of 50 m BF ron + W = Fy of water. Determine the stress in the steel fy = 739.66 - 50 ee 8 Solution Froit = 86.2 kN D [S; = al * 9,81(50) (300 Problem 3 - 48 Sie a = 6131.25 kPa Determine the force F S, = 6.13 MPa required to hold the cone shown. Neglect the weight of the cone Problem 3 - 50 Determine the required thickness of a 450 mm diameter steel pipe to carry a maximum pressure of 5500 kPa if the allowable working stress of steel is 124 MPa e Solution pD Co he bee 45 124 x 1000 = me t= 9.98 mm say 10 mm FLUID MECHANI & HYDRAULIGS Problem 3 - 51 Determine the stress at the walls of a 200 mm diameter pipe, 10 mm i. under a pressure of 150 m of water and submerged to a depth of 20 m in sali water 152 CHAPTER THREE Total Hydrostatic Force on Surfaces Solution Ges [s= F | P = Pinside ~ Poutside p = 9.81(150) - 9.81(1.03) (20) p= 1269.4 kPa = 1,269 MPa 4 DEE2O0) sap60 MPa 2(10) Problem 3 - 52 A 100-mm-ID steel pipe has a 6 mm wall thickness. For an allowable tensile stress of 80 MPa, what maximum pressure can the pipe withstand? Solution a p(100) 2(6) p =9.6 MPa = 9,600 kPa 80 Problem 3 - 53 A wooden storage vat is 6 m in diameter and is filled with 7 m of oil, s = 0.8 The wood staves are bound by flat steel bands, 50 mm wide by 6 mm thick, whose allowable tensile stress is 110 MPa. What is the required spacing of the bands near the bottom of the vat, neglecting any initial stress? Solution 2 pD Allowable tensile stress of hoops, $, = 110 MPa Cross-sectional area of hoops, A; = 50(6) = 300 mm? Spacing of hoops, 5 = FLUID MECHANICS & HYDRAULICS CHAPTER THREE 153 Total Hydrostatic Force on Surfaces Pipe diameter, D = 6 m = 6000 mm Maximum pressure the tank (at bottom), p = youl p = 9.81(0.8)(7) = 54.936 kPa _ 2(110 x 10*)(300) 54.936(6000) 5 = 200.23 mm say 200 mm Problem 3 - 54 A thin-walled hallow sphere 3.5 m in diameter holds helium gas at 1700 kPa. Determine the minimum wall thickness of the sphere if its allowable stress is 60 MPa. Solution pD 1 stress, S.= —— Wall stress, S; ri (3. 000 60,000 = 1,700(3.5 x 1000) 4t t= 24.79 mm ’ Problem 3 - 55 A vertical cylindrical tank is 2 meters in diameter and 3 meters high. Its sides are held in position by means of two steel hoops, one at the top and the other at the bottom. If the tank is filled with water to a depth of 2.1 m, determine F the tensile stress in each hoop. Solution

You might also like